Download A 77-year-old man comes to the office with his wife because of

Survey
yes no Was this document useful for you?
   Thank you for your participation!

* Your assessment is very important for improving the workof artificial intelligence, which forms the content of this project

Document related concepts

Medical ethics wikipedia , lookup

Patient safety wikipedia , lookup

Adherence (medicine) wikipedia , lookup

Dysprosody wikipedia , lookup

List of medical mnemonics wikipedia , lookup

Dental emergency wikipedia , lookup

Transcript
A 77-year-old man comes to the office with his wife because of "walking difficulties." He
says that over the past 5 months he has noticed that when he walks or stands for longer
than 15 minutes he gets pain and weakness in his thighs. The pain is usually relieved by
sitting. Within the past 1-2 years he began to get a "discomfort" in the anterolateral
thighs, more in the right lower extremity than the left. He also gets a pain in his right hip,
which radiates down to just below his knee. He denies ever having any calf pain. He
urinates 2-3 times per night and will lose 1 or 2 drops of urine if he cannot make it to the
bathroom in time. His wife has notice that he has a tendency to stand with his knees
slightly bent rather than straight legged. He tells you that 16 years ago he began to feel
"unsteady on his feet." He did not fall or experience pain at that time, but he had "pins
and needles feelings" in his fingers and feet and "lost the feeling of his feet being
attached to the ground." He saw 2 different doctors at that time, had a myelogram, and
was diagnosed with C4-5 damage. He underwent C4-5 intercervical discectomy and
osteophyte removal. After the surgery he wore a neck brace for several months and the
symptoms remained stable. He noticed that his knee reflexes were stronger after the
surgery. 4-5 years ago he began to notice that his right knee would buckle. This resulted
in 2-3 falls over a 1-year period. He saw a neurologist who prescribed physical therapy
and a cervical collar to be worn at night. He did well and stopped wearing the collar
about 1 year ago. Physical examination shows weak, but palpable distal pulses,
moderately limited neck range of motion, mild weakness of the deltoids and biceps
bilaterally, mild weakness of hamstrings and extensor hallucis longus bilaterally, and a
normal sensory exam. He has brisk symmetric deep tendon reflexes and down-going
toes bilaterally. Tests of coordination are normal and his gait is normo-based and steady
but mildly spastic. Cranial nerve and mental status examinations are unremarkable. The
most appropriate next step is to
A. order an MRI of the brain
B. order an MRI of the cervical spine
C. order an MRI of the lumbar spine
D. perform electromyography and nerve conduction study
E. send him for an angiogram of the lower extremities
Explanation:
The correct answer is C. The history is typical of that for neurogenic claudication, which
is caused by lumbar stenosis. The lower extremity weakness is also typical of bilateral
L4/L5 radiculopathies. You would want to do an MRI to confirm the diagnosis and make
sure that other pathology such as a spinal tumor was not responsible.
There is no reason to suspect brain pathology in this case (choice A). If there were
cranial nerve or mental status abnormalities it should be considered.
His gait disorder is being caused primarily by pain, which is secondary to neurogenic
claudication of the lumbar spine. He has known cervical spine problems, which may be
contributing by impairing balance. It would also cause weakness of deltoids and biceps
by C5 root compression. He may need a cervical spine MRI (choice B) at some point,
but it is not primarily causing his walking troubles.
There is no indication for electromyography or nerve conduction (choice D) as the
history and exam are sufficient to make this diagnosis. An MRI should be ordered to
confirm the diagnosis and rule out other pathology.
1
An angiogram of the lower extremities might be indicated if you suspected vascular
claudication (choice E). More commonly this would involve the calves and distal pulses
would not be palpable.
A 29-year-old woman comes to the clinic complaining of thumb pain. She tells you that
she sustained a fracture of the distal radius from a cycling accident 6 weeks ago. At that
time a comminuted dorsally displaced fracture of the distal radius was diagnosed by xray of the wrist and a cast was applied. The cast was removed last week and the patient
resumed normal activity as a schoolteacher. The pain has gradually gotten worse, and
radiates from the base of the thumb through the palm. Physical examination reveals pain
of the anatomic “snuff box". There is normal sensation and motion of the hand. Pain
increases slightly with hand or wrist motion, but the grip is strong. There is a normal
radial pulse. Repeat wrist x-ray reveals a healed Colles fracture and an incidental note is
made of sclerosis of the scaphoid (navicular) bone. The most appropriate next step in
the management of this patient is to
A. order a repeat x-ray of the hand
B. perform compartment measurements to evaluate for compartment syndrome
C. perform diagnostic arthrocentesis of the first carpometacarpal joint
D. perform Tinel and Phalen tests to evaluate for carpal tunnel syndrome
E. request an urgent orthopedic surgery consult to evaluate for avascular
necrosis
Explanation:
The correct answer is E. This patient has avascular necrosis (AVN) of the scaphoid
(navicular) bone. This is a recognized complication of hand/wrist trauma. Fractures of
the waist of the scaphoid bone compromise blood supply to the bone leading to AVN
which produces pain in the anatomic “snuff box” and sclerosis of the scaphoid bone on
plain radiograph. Scaphoid fractures are often occult on plain radiographs and, hence,
repeat radiographs should be performed if there is “snuff box” pain after hand or wrist
trauma. Surgery must be performed to pin the scaphoid together and allow healing of
the bone and blood vessels.
The diagnosis of AVN has been established by the history, physical exam, and
radiograph results so repeat films are not necessary (choice A). A surgical consult is
needed.
Compartment measurements are necessary in the case of a suspected compartment
syndrome (choice B). The clinical indicators of compartment syndrome are extremity
pain, pulselessness, paraesthesia, and pallor indicating a “perishing” extremity.
Arthrocentesis (choice C) is necessary if infection is suspected. The absence of fever
argues against infection. Infected joints may be secondary to direct trauma such as an
open fracture, but present within days of the initial injury. Hematogenous infectious
seeding of a joint occurs in the setting of sepsis. Arthrocentesis must be done urgently
in this setting to allow for antibiotic treatment to avoid joint destruction.
Tinel and Phalen tests (choice D) are used to assess median nerve function in the
setting of carpal tunnel syndrome (CTS). CTS is a chronic condition involving the
compromise of the median nerve in the carpal tunnel. It is not associated with trauma,
but repetitive wrist activity such as typing.
2
A 87-year-old man with a long-standing history of smoking, chronic obstructive lung
disease, peripheral vascular disease, and multiinfarct dementia is admitted to the
hospital under your care for gangrenous involvement of the second and fourth toe of his
right foot. An arteriogram shows that revascularization is not feasible. After extensive
evaluation the surgeon recommends amputation below the knee. The patient is alert and
oriented to person, place, and situation. He has some in short-term memory deficits and
higher cognitive functions seem intact. You and the surgeon explain the situation to the
patient, and despite his dementia, you both believe that he adequately understands the
risks and benefits of the surgery. Another physician and nurse also witness the consent
procedure and believe that the patient has full comprehension of the risks and benefits
of the surgical procedure. However, the patient's son, who is the designated proxy in the
patient's power of attorney health care documents, adamantly opposes any surgical
intervention. He points out that the patient has indicated in the document that he would
not like any aggressive measures to save his life. Despite detailed discussions and
several family meetings, the son remains adamant about refraining from any surgical
intervention. The most appropriate action is to
A. arrange for a psychiatry referral to assess patient's competency
B. ask the son to find other physicians to care for his father
C. consult your hospital attorney
D. discharge the patient form hospital
E. proceed with a below-the-knee amputation
Explanation:
The correct answer is E. Clearly the most appropriate line of management at this time is
the below-the-knee amputation. The patient is competent and has given his informed
consent.
A psychiatry consultation (choice A) is not necessary in this case. Although the patient
has some dementia with some memory deficits, 3 physicians and a nurse think him
capable of making his own decisions. Even though the patient has indicated in his
advance directive that he would not like any “aggressive measures,” this does not go
into effect until the patient loses complete decision making capacity.
Asking the son to find another physician to care for his father (choice B) is another
option, but the son has no legal standing in this situation, since he is disagreeing with
his father's wishes.
Consulting the hospital attorney (choice C) is not necessary as there is no legal issue
for the hospital.
Discharging the patient from hospital (choice D) would be unethical..
A mother brings her 3-month old son to the emergency department stating that "for the
past 2 days the infant has been moving the right lower extremity less than the left leg".
She is concerned that the right hip may be dislocated even though she denies any
history of trauma. She tells you that the pregnancy was non-complicated and a
3
scheduled cesarean section was completed at 34-weeks gestation secondary to twins.
The other twin's past medical history is significant only for hyperbilirubinemia at birth that
resolved without complication after 4 days of phototherapy. Physical examination reveals
a well-nourished and developed 3-month-old boy with focal tenderness at the distal right
thigh. Funduscopic examination is negative. There are no skin lesions, no regions of
ecchymosis, abrasions, or other skin changes. The right thigh and knee have no gross
deformity, no signs of trauma. Passive hip range of motion is full and symmetric
bilaterally. Pelvis and lower extremity x-rays reveal a 2 mm fracture at the medial
metaphysis of the right distal femur. The fracture is minimally displaced. There are no
other signs of fracture, dislocation, or other bony pathology. When you explain the
results to the mother, she is relieved to know the hip is not dislocated. She states that
she needs to get home to her other children and husband. The best course of
management at this time is to
A. inform the mother that the fracture is relatively insignificant and will heal
uneventfully. Splint the right leg, give an orthopaedic surgery follow-up appointment
for 1 week, and discharge the patient
B. inform the mother that the fracture is relatively insignificant and will heal
uneventfully. The fracture measures only 2mm and is minimally displaced so no
splint required. Discharge the patient with a 1-week follow-up appointment in the
clinic
C. complete a skeletal survey and contact the patient's pediatrician by telephone
to inquire if there is any history of child abuse in this family. If the pediatrician relays
no significant history and the skeletal survey is negative, then discharge the patient
home. Splint the right leg and give a follow-up appointment in the clinic
D. complete a skeletal survey. If there are no other signs of acute or healed
fractures, then splint the right leg and discharge to home
E. complete a skeletal survey and contact child protective services (CPS). Make
arrangements for admission until child protective services completes their
evaluation. If necessary, call security and detain the child against the mother's
consent
Explanation:
The correct answer is E. Child abuse can occur in any family. A high index of suspicion
may save a child from further abuse, or even death. It should be suspected in any and
all scenarios. A medial or lateral metaphyseal fracture of any long bone is classic for
child abuse. These fractures are referred to as "metaphyseal corner" fractures. More
importantly, this 3-month-old boy probably does not roll over let alone bear any weight
in the lower extremities. The child could not have fallen on his own to induce this
fracture. Any child who is not walking and has a lower extremity fracture is a victim of
child abuse until proven otherwise. There are definitely mechanisms of accidental
trauma that could cause this injury. This is a diagnosis of exclusion. Although the
physical exam is negative for other injuries, a skeletal survey must be completed. It is
not an option, it must be performed. The literature varies in the reported rate of
uncovering other injuries. Do not be the physician who is deceived by the history,
physical examination, or the family's social class. Always suspect child abuse,
document a complete physical exam, and contact CPS.
To inform the mother that the fracture is relatively insignificant and will heal
uneventfully, splint the right leg, give an orthopaedic surgery follow-up appointment for
1 week, and discharge the patient (choice A) is correct except for discharging the
4
patient home. CPS must be contacted in the ER. No growth disturbances are expected.
Inform the mother that the fracture is relatively insignificant and will heal uneventfully
since the fracture measures only 2 mm and is minimally displaced and that no splint is
required. Discharge the patient with a 1-week follow-up appointment with the
pediatrician (choice B) is incorrect. Always splint long bone fractures. They may
progress if not protected. At the very least a splint reminds caregivers of the fracture.
CPS must be contacted in the ER.
Complete a skeletal survey and contact the patient's pediatrician by telephone to
inquire if there is any history of child abuse in this family. If the pediatrician relays no
significant history and the skeletal survey is negative, then discharge the patient to
home. Splint the right leg and give a pediatric follow-up appointment (choice C) is
incorrect. Lack of a history of child abuse is irrelevant. The patient presents with an
injury consistent with abuse. One episode warrants for CPS to be involved. Laws vary
by state, but it is safe to assume that all states require all suspected cases of child
abuse to be reported to CPS.
This child cannot be discharged before CPS consultation (choice D). Admission should
be through any accepting service, orthopaedic surgery, pediatrics, etc. If this patient is
discharged, the next visit to the ER may be for a much more significant injury or even
death.
A 51-year-old woman who your partner has been treating for low back pain calls the
office complaining of a change in symptoms. She reports that although she has had
intermittent low back pain for a number of years, in the past few weeks there has been a
profound increase in her pain. She also reports that her left leg often tingles and is
numb. In reviewing her records, it is noted that the patient has been seen in the office
over the years for back pain and has recently been informed that she can no longer
obtain narcotics. The patient works as a daycare manager and often lifts small children.
She is married with a 4-year-old daughter. The most appropriate next step is to
A. ask the patient to come to the office for a neurological evaluation
B. order an MRI of her lumbar spine
C. prescribe additional narcotic analgesia
D. refuse to see the patient as she is malingering
E. schedule the patient for epidural steroid injections
Explanation:
The correct answer is A. This patient suffers from low back pain. This entity is one the
most challenging disorders in all of medicine. It has been estimated that 70% of the
population suffers from some sort of low back pain at an annual cost of between $20
and $50 billion. The etiology is often unknown in 85-90% of cases but most cases
resolve with support and mild pain relief. The goals of evaluation are primarily to
exclude serious underlying visceral or systemic disorders. This patient, despite being
seen for back pain in the past, has new neurological symptoms which suggest an
underlying etiology that is not simply musculoskeletal. She requires a full physical
examination.
5
In the absence of a full exam, which would point to a specific diagnosis, it is not
appropriate to order an MRI of her lumbar spine (choice B) without seeing the patient.
In the presence of new signs or symptoms such as the ones this patient is describing,
masking the pain with narcotics (choice C) is incorrect. In addition, narcotics have been
shown to be only minimally effective in managing low back pain and are not considered
standard management for this condition.
Even though this patient has had a complex past history with the clinic, her new signs
and symptoms point towards a specific diagnosis such as spinal stenosis or disk
herniation and she requires evaluation. To consider her a malingerer (choice D) without
evaluation is inappropriate.
Epidural steroid injections (choice E) is a specific therapy aimed at reducing
inflammation in certain conditions such as epidural adhesions, facet degeneration, and
refractory low back pain. Their efficacy is uncertain and the treatment is usually
attempted only after more serous disease is excluded and the patient has failed
physical therapy and oral antiinflammatory agents.
A 67-year-old man comes to the clinic with a new rash on his upper eyelids that he says
he has had for a few months. He also reports to have experienced increasing weakness
of his lower extremities, especially when he tries to stand up from a sitting position. He
has hypertension, which is well controlled with amlodipine, and hypercholesterolemia
controlled with diet and exercise, that he's no longer able to perform. He denies any
recent weight loss. Physical examination reveals a well-nourished male with normal
chest and lung exam. A diffuse, ill-defined, violaceous plaque is apparent on each upper
eyelid. The oral mucosa is within normal limits. Notable on extremities are firm,
violaceous plaques overlying the proximal and distal interphalangeal joints on the
dorsum surface. There is no involvement of the space between joints on his hands. The
general appearance of his hands resembles a mechanic's hands. He also has a
poikiodermatous patch (hypopigmentation, hyperpigmentation, telangiectatic, and
atrophy) involving the V of the neck as well as the upper back. Brief neurologic
examination reveals proximal lower extremity weakness. The most appropriate
management of this patient is to
A. determine the antinuclear antibody level
B. determine anti-Rho and anti-La levels
C. order a liver function test
D. order a rheumatoid factor level
E. search for an internal neoplasm
Explanation:
The correct answer is E. Searching for internal neoplasm is the best work up, because
dermatomyositis in adults often appears as a paraneoplastic phenomenon. Progressive
weakness is the major clinical manifestation. Difficulty in rising from low chairs, climbing
stairs, or holding the arms above the head are common symptoms. The most specific
skin manifestation is a maculopapular erythema on the bony prominences, such as the
knuckles, elbows, and knees that appears in about 70% of cases. A blotchy erythema,
6
especially on sun-exposed skin, occurs in about a third of patients. More chronic
macular erythema and poikilodermatous lesions may also develop on the trunk and
proximal parts of the limbs. A distinctive sign is an erythematous blush on the eyelids
and around the eyes. It has a lavender shade and is called a heliotrope erythema.
Patients over the age of 50 at onset, should be evaluated for the presence of a
malignant tumor. Chest x-ray, gastrointestinal evaluation, and careful breast and
genitourinary examinations are sufficient unless symptoms point elsewhere.
While antinuclear antibody (choice A) may be positive in dermatomyositis, it is not the
best work up nor is it the most diagnostic test.
Anti-Rho and anti-La (choice B) is incorrect, because these are the main markers for
Sjogren's, subacute lupus erythematosus, and neonatal lupus, but not for
dermatomyositis.
A liver function test (choice C) is incorrect, because it is neither diagnostic for
dermatomyositis nor the best work up.
Checking rheumatoid factor levels(choice D) is incorrect, because dermatomyositis
patients do not usually have an elevated rheumatoid factor level.
A 23-year-old woman comes into the office because of a 1-week history of painful
nodules on her legs. She was recently seen in your office and started on oral
contraceptives for the first time. About 2 weeks earlier, she had a "cold" which was
associated with sore throat and fever, but she recovered without complications. Physical
examination shows multiple red, deep-seated, tender, 2-4 cm nodules on the pretibial
region of both legs. She says that "the first lesions" have flattened, leaving a purple or
blue-green discoloration. No involvement of the ankles or posterior legs is appreciated.
The most appropriate management of her condition is to
A. administer penicillin IM for the presumed upper respiratory infection
B. admit her to the hospital and begin amphotericin for presumed deep fungal
infection
C. change her to a different oral contraceptive pill
D. prescribe acetaminophen with codeine
E. recommend ibuprofen 600mg 4 times per day
Explanation:
The correct answer is E. Ibuprofen or another nonsteroidal antiinflammatory drug is the
treatment of choice for erythema nodosum (EN). EN is a common inflammatory
panniculitis which occurs in two forms: acute or chronic. Acute EN most commonly
occurs in young adult women with bilateral, symmetrical, deep, tender nodules 1-10 cm
in diameter. Usually there are up to 10 lesions but many more may be found. The most
common location is the pretibial area. The onset is acute, frequently associated with
malaise, leg edema, and arthritis or arthralgias (usually of the ankles, knees or wrists).
Over a few days, the lesions flatten, leaving a purple or blue green color resembling a
deep bruise. It is commonly associated with a streptococcal infection. Other infectious
etiologies have been implied including Mycobacteria tuberculosis, Yersinia, Shigella,
Salmonella, or fungal infection. Drugs may also induce erythema nodosum including
bromides, iodides, and sulfonamides. The most common drug to induce EN is oral
7
contraceptives. Besides treatment with NSAIDs, if lesions do not resolve within weeks
of treatment, physicians should investigate for other infectious causes. Discontinuing
oral contraceptive pills may also be important.
Penicillin IM (choice A) is incorrect because treating a respiratory infection without
positive culture is not the standard of practice. The proper step here would have been
performing a throat culture to determine if Streptococcus is the offending organism prior
to initiating treatment
Admit to the hospital to start amphotericin (choice B) is incorrect because deep fungal
infection does not usually appear in crops in a healthy, immunocompetent patient. This
patient is classic for either streptococcal or drug-induced EN.
Changing oral contraceptive (choice C) is incorrect because most oral contraceptives
have been associated with development of EN. Rather than changing to a different type
of contraceptive, this patient may be better off discontinuing OCPs.
Acetaminophen with codeine (choice D) is incorrect because only NSAIDs or
indomethacin are helpful in EN.
You are seeing an 83-year-old woman for preoperative clearance prior to a total hip
replacement. She has a long history of rheumatoid arthritis with pulmonary involvement.
Her disease has, however, been well controlled over the last several years on
methotrexate. She has no known allergies to any medications. She does not smoke or
drink alcohol. She is active and walks a mile, 3 times a week. Her temperature is 37.7C
(99.9 F), blood pressure is 110/56 mm Hg, pulse is 89/min, and respirations are 19/min.
Her hands show degenerative changes consistent with long standing rheumatoid
disease. Her lungs are clear to auscultation and her cardiac rhythm is regular. She has
limited neck flexion-extension. A neurologic examination is unremarkable. A chest
radiograph is unremarkable. An electrocardiogram shows a sinus rhythm with some nonspecific ST and T wave abnormalities. During your discussion, the patient expresses a
desire to have a general anesthetic. Based upon the available information, the next most
reasonable imaging study to obtain is
A. a cervical spine radiograph
B. a chest CT
C. a head CT
D. a spine MRI
E. an echocardiogram
Explanation:
The correct answer is A. Since the patient is insisting on a general anesthetic and will
thus require intubation, it is important to ensure that the patient has no cervical disease
as a result of her rheumatoid arthritis, especially given her history of limited neck
mobility.
A chest CT (choice B) in the setting of a benign lung exam and chest radiograph is
unlikely to add useful information in this scenario.
A head CT (choice C) is not indicated in this setting, since there is no reason to suspect
any intracranial pathology.
8
A spine MRI (choice D) is not indicated, given the absence of any historical or physical
exam features to suggest spinal cord pathology.
An echocardiogram (choice E) is not indicated given that she is very active and has an
unremarkable cardiac exam and electrocardiogram.
A 28-year-old man comes to the emergency department because of moderate left
shoulder pain that is worse with abduction of the shoulder. He plays baseball
occasionally with friends and has noticed that the pain worsens when throwing the ball.
He denies any history of trauma. Physical examination shows weakness of the shoulder,
most pronounced with abduction. A shoulder x-ray reveals no fractures or dislocations.
The most appropriate next step in management is to
A. advise patient that he should rest and ice the shoulder
B. order an emergent CT scan of the shoulder
C. order an emergent MRI of the shoulder
D. order an outpatient MRI of the shoulder
E. prescribe nonsteroidal antiinflammatory drugs
Explanation:
The correct answer is D. Rotator cuff tears are chronic use injuries most common in
throwing athletes. The clinical symptoms are weakness and instability of the shoulder.
An x-ray may show a subacromial spurring, a high-riding shoulder, or calcific tendonitis.
Diagnosis is made by MRI of the shoulder. This is a non-emergent diagnosis and the
imaging can be performed on an outpatient basis. Repair is made arthroscopically.
Advising the patient to rest and ice the shoulder (choice A) will help alleviate the
immediate symptoms of pain. It will not, however, treat the underlying condition of a
rotator cuff tear. Rest and ice are good for muscle sprains, but will not help torn
tendons.
A CT scan of the shoulder (choice B) is not indicated in suspected cases of rotator cuff
tears. A CT is good for evaluating the bones of the shoulder, but does not provide the
resolution to evaluate the muscles and tendons of the shoulder.
An emergent MRI of the shoulder (choice C) is not indicated. Rotator cuff tears are
chronic overuse injuries and do not require emergent diagnosis or repair. The MRI
should be ordered on an outpatient basis.
Prescribing nonsteroidal antiinflammatory drugs (choice E) will help alleviate the
immediate symptoms of pain. It will not, however, treat the underlying condition of a
rotator cuff tear.
A 35-year-old woman is in the hospital for a flare of nephritis related to systemic lupus
erythematosus (SLE). On rounds in the morning, she complains of right hip pain. She
states that for the last several weeks, she has had a deep aching in the hip and now it is
getting much worse since she was in the hospital. It hurts her both at rest and with
motion. She denies any history of trauma, and has not started any new activities. There
have been no fevers. Her only outpatient medication is prednisone 10 mg daily, and she
9
takes ibuprofen for pain relief, which has helped minimally. However, now she is on a
higher dose of intravenous steroid. Her temperature is 37.2 C (99 F), blood pressure is
132/82 mm Hg, and pulse is 72/min. There is no pain on palpation over the hip but pain
is present with range of motion. Laboratory studies show a leukocyte count 8,100mm3
and a hematocrit 34%. A plain x-ray of the pelvis and hip is normal. The next most
appropriate step is to
A. assure her that her pain will resolve spontaneously
B. continue the patient on oral ibuprofen and follow her symptoms as an
outpatient in 1 month
C. increase the duration of her intravenous steroids
D. order an MRI of the hip
E. send her for an arthrogram of the hip
Explanation:
The correct answer is D. In a patient with nontraumatic hip pain, who is on chronic
steroid therapy for a disease such as SLE, a major concern is avascular necrosis (AVN)
of the bones. AVN passes through several stages, the first of which has a totally normal
x-ray. It will progress to patchy sclerosis, then subchondral fracture, cortical collapse,
and eventually lead to severe irreversible degenerative change requiring total hip
replacement. In the setting of AVN, an MRI will show marked bone marrow edema,
even when the x-ray is totally normal and is the only objective test to rule out the
disease in this early stage. Treatment early can prevent disease progression. Surgical
core decompression is one potential treatment, where a core of bone is taken from the
femoral head and replaced with a vascularized fibular graft. Trials are also ongoing to
evaluate the effectiveness of packing the core with bone morphogenic proteins to
promote healing.
Assuring her that this will resolve spontaneously (choice A) or following up as an
outpatient in 1 month (choice B) are not good options. The longer this disease goes
unrecognized, the greater chances are that it will progress to the more advanced
stages.
Systemic corticosteroids are actually the underlying cause of the AVN rather than the
SLE itself. In other diseases, such as sickle cell, the disease itself, actually results in the
AVN. In any case, increasing the dose of prednisone (choice C) would actually
exacerbate the problem.
An arthrogram of the hip (choice E) would not be useful in this diagnosis, as it will
evaluate the joint space, but will not add any information about what is going on within
the bone marrow.
A 67-year-old man comes to the clinic with a lesion on his lower lip. He tells you that this
lesion started out as a “pimple” and has been increasing in size over the past 8 months.
He has been smoking 2 packs of cigarettes a day for the past 40 years. Physical
examination shows an ulcerated 0.9 cm nodule on the lower lip. There are multiple
small, firm, non-mobile lymph nodes in the cervical region. He appears to have sun
damage on his face. At this time the most correct statement about his condition is:
10
A. His history of sun exposure is more important than smoking in the
development of this lesion
B. This is most likely basal cell carcinoma of the lip requiring lip wedge resection
C. This is most likely leukoplakia of the lip
D. This is most likely squamous cell carcinoma and a referral for mass and lymph
node resection followed by hematology/oncology evaluation is indicated
E. Squamous cell carcinoma of the lip has the same prognosis as squamous cell
carcinoma of the face
Explanation:
The correct answer is D. This is most likely squamous cell carcinoma (SCC) and a
referral for mass and lymph node resection followed by hematology/oncology evaluation
is indicated. SCC of the lip is as aggressive as SCC of oral mucosa and is commonly
associated with smoking, smokeless tobacco, betel nut chewing, and alcohol
consumption. It occurs primarily in older men. The most frequent sites are the lower lip,
tongue, soft palate, and the floor of the mouth. Unfortunately, the survival rate is only
30% because squamous cell carcinoma is often discovered late, often it has
metastasized to the cervical lymph nodes. The best approach for this patient is referral
to otolaryngology for mass and neck resection and then followed by adjuvant radiation
or chemotherapy as determined by an oncologist.
History of sun exposure is more important than smoking (choice A) is incorrect. Oral
SCC can be associated with actinic damage, but tobacco and alcohol are the most
common risk factors for head and neck cancers.
Basal cell carcinoma (choice B) is unlikely because these lesions tend to have soft,
friable and pearly appearances with telangiectasias.
Leukoplakia of the lip (choice C) is incorrect as this term refers to whitish plaques or
patches in mucous membranes, which may or may not show cellular atypia.
Squamous cell carcinoma of the lip has the same prognosis as ones of the face (choice
E) is incorrect. The prognosis of SCC on face (i.e. forehead) is excellent when caught
early and complete resection has a greater than 85% cure rate, whereas, oral SCC has
poor prognosis in general and grows much more rapidly.
A 64-year-old man comes to the clinic because of a "spot" on the side of his face that
has been there for about 8 months. He says that he is an executive at a local company
and is retiring at the end of the year. The company has hired a portrait artist to paint his
picture that will hang in the boardroom for many years to come, and so he realized that
this is a good time to "have this thing taken off." He thinks that the lesion has not grown
since he noticed it, but he has not paid it much attention. He plays tennis every weekend
at his country club and then lies in the sun with his wife. This is the first time you have
seen this patient, but he tells you that he has been very healthy and has only suffered
through "a couple of bouts of kidney stones" over the years. Physical examination shows
a 2.3-cm waxy, verrucous, dark brown papule with a “stuck-on” appearance.The most
likely diagnosis is
A. actinic keratosis
B. basal cell carcinoma
11
C. dermatofibroma
D. melanoma
E. psoriasis
F. seborrheic keratosis
G. squamous cell carcinoma
Explanation:
The correct answer is F. This patient most likely has seborrheic keratosis. This lesion is
characterized by light brown to black papules or plaques with an adherent waxy, greasy
scale. The "stuck-on" appearance is very characteristic. It is most often found on the
face and trunk.
Actinic keratosis (choice A) is characterized by flesh-colored or red to brown macules or
papules with a rough scale. It is most common on sun-exposed skin.
Basal cell carcinoma (choice B) is characterized by a papule with a central ulceration
and a pearly, raised border. It is most common on sun-damaged skin.
Dermatofibroma (choice C) is characterized by a firm, red to brown nodule with a
dimpling of the skin. It can occur anywhere.
Melanoma (choice D) is characterized by an asymmetric, pigmented lesion with an
irregular border. Any change in an existing skin lesion should be evaluated for
melanoma.
Psoriasis (choice E) is characterized by erythematous plaques and papules covered
with silvery scales. The most common sites are elbows, knees, scalp, and back. Pitting
of the nails may be present.
Squamous cell carcinoma (choice G) is characterized by a hyperkeratotic, ulcerated, or
crusty lesion. The face, especially the lower lip, is the typical site of involvement.
A 15-year-old boy who has been HIV-positive since birth is admitted to the hospital
because of severe lower back pain. He denies a history of trauma and “has no idea how
this started". His temperature is 37 C (98.6 F), blood pressure is 140/70 mm Hg, pulse is
100/min, and respirations are 19/min. Physical examination shows point tenderness of
L4 and L5 posteriorly. A neurologic examination is otherwise unremarkable and there is
a normal gait. Laboratory studies show a leukocyte count of 15,000/mm3. An MRI of the
lumbar spine reveals inflammation of the L4 and L5 vertebral bodies and the L4-5
intervertebral disc. There is a focal fluid collection in the L4-5 disc space. There is no
cord compromise or abscess. The next step, after initiating broad-spectrum antibiotic
therapy is to
A. aspirate the L4-5 disc space under fluoroscopic guidance
B. prepare him for an exploratory laminectomy and decompression
C. order lower extremity nerve conduction studies
D. send him for a non-contrast head CT
E. send STAT blood cultures
12
Explanation:
The correct answer is A. This patient has symptoms and imaging findings consistent
with L4-5 discitis. Given the immunocompromised status of the patient, a wide variety of
organisms are likely pathogens. In this case, hematogenous spread to the disc space is
likely. It is essential to begin immediate broad-spectrum antibiotic therapy and obtain
tissue in an attempt to isolate an organism. Systemic blood cultures will certainly be
drawn, but they are low yield. Tissue must be obtained under imaging guidance to
isolate an organism and narrow antibiotic coverage.
Exploratory laminectomy and decompression (choice B) would be necessary only in the
setting of an abscess that compromises the spinal cord. A repeat MRI should be
performed if symptoms of cord compression develop.
Lower extremity nerve conduction studies (choice C) are used in the setting of
suspected peripheral neuropathy. The physical exam is normal in this case, so there is
no indication of spinal cord compression at this time.
Non-contrast head CT (choice D) is necessary prior to lumbar puncture to exclude a
cause of increased intracranial pressure that could lead to herniation during or after a
lumbar puncture. A lumbar puncture is not necessary here given the lack of neurologic
and MRI findings of cord compromise. Blood cultures (choice E) must be drawn, but
they are unlikely to be positive given the lack of fever. Obtaining tissue for staining and
culture, at the infected site, is paramount.
A 55-year-old man comes to the emergency department because of numerous pustules
associated with fever, fatigue, and arthralgias over the past 2 days. He has a history of
"eczema" and was seen by a dermatologist 10 days prior due to a "flare-up of his
eczema." He was given tapering doses of oral corticosteroids, which he completed 3
days ago. On further questioning you learn that his father and brother both have severe
psoriasis. His temperature is 38.8 C (101.8 F). He has dry oral mucosa and generalized
erythema with hundreds of small, pinpoint pustules involving the entire skin surface,
some coalescing into large pools of pustule pockets. There are multiple "oil drops" of
pitting in the fingernail beds. At this time the most correct statement is that
A. the appropriate management would be to culture the pustules and admit the
patient for intravenous antibiotic therapy
B. he has pustular psoriasis and the offending culprit is the systemic
corticosteroids
C. he has scalded skin syndrome and oral antibiotic therapy is indicated.
D. he has toxic epidermal necrolysis and you need to start intravenous
immunoglobulin treatment
E. this is a pustular drug eruption; the patient is allergic to prednisone
Explanation:
The correct answer is B. Pustular psoriasis is characterized by acute onset of sheets of
sterile pustules on a background of erythema. The pustules often become confluent,
forming lakes of pus. The skin is painful and tender. The pustules are accompanied by
13
fever, malaise, arthralgia, and diarrhea. Pustular psoriasis in known to be triggered by
systemic corticosteroids as a rebound phenomenon in a person with known psoriasis.
In this patient, his previously treated "eczema" may have been a misdiagnosed psoriatic
plaque, given his strong family history of psoriasis. Treatment includes supportive care,
intravenous hydration, and systemic cyclosporine.
To culture the pustules and admit for intravenous antibiotic (choice A) is incorrect
because as mentioned above, pustular psoriasis is composed of sterile pustules and an
antibiotic will not alleviate the flare.
Scalded skin syndrome (choice C) is incorrect because this is a disorder generally seen
in adults with immunosuppression or renal impairment. Clinical manifestation includes
malaise, fever, irritability, a generalized macular erythema, and a fine, stippled,
sandpaper or nutmeg-like appearance that quickly progresses to a tender scarlatiniform
phase over 1 to 2 days. From the intertriginous and periorificial areas and trunk, the
erythema and tenderness spread over the entire body, usually sparing the hairy parts.
This is followed by an exfoliative phase, heralded by exudation and crusting around the
mouth and sometimes the orbits.
Toxic epidermal necrolysis (choice D) is incorrect because this is a disorder with
widespread macular erythema and stomatitis with exfoliation of the skin. As the
erythema becomes more universal, the skin begins to develop areas of separation
between epidermis and dermis. The pathogenesis of toxic epidermal necrolysis involves
a drug-induced necrosis of the basal cell layer of the epidermis, with the production of
subepidermal separation rather than a superficial split as seen in scalded skin
syndrome.
Pustular drug eruption (choice E) is incorrect because prednisone can cause acneiform
eruption at follicles but does not usually cause pustules that coalesce into lakes. In
addition, the strong family history of psoriasis in this patient should lead you to a
diagnosis of pustular psoriasis.
A 32-year-old gardener comes to the office because of a lesion on his nose that has
been increasing in size over the past few weeks. He says that he rarely sees a doctor
and is reluctant to discuss his medical history. He works outside for 9 to 10 hours a day,
smokes two packs of cigarettes a day, and drinks approximately a case of beer a night.
He states that he sometimes "shoots up" heroin with a few guys that "hang out on a
stoop around the block." Physical examination shows a 1.5-cm, purple nodule on the tip
of his nose, a 0.6-cm, reddish-purple raised macule on the tragus of his left ear, a 1-cm,
purplish-blue area of discoloration on the roof of his mouth, and a 4-cm, confluent lesion
made up of purplish-brown plaques, patches, nodules, and tumors on the anterior
surface of his right leg. The results of the punch biopsy, which return in one week, show
spindle cells, endothelial cells, and the extravasation of red blood cells. The factor in this
patient's history most closely correlated with these lesions is
A. alcohol intake
B. cigarette smoking
C. intravenous drug use
D. pesticide exposure
14
E. sun-exposure
Explanation:
The correct answer is C. This patient most likely has Kaposi's sarcoma, which is one of
the most common malignancies affecting HIV-infected individuals. It is a vascular tumor
characterized by purplish lesions (macules, plaques, nodules, and tumors) that occur
most commonly on the face, oral mucosa, lymph nodes, lungs, and gastrointestinal
tract. Biopsies of the lesions show spindle cells, endothelial cells, and the extravasation
of red blood cells. Management of Kaposi's sarcoma is dependent on the size, location,
and the severity of other diseases, and includes observation, surgery, radiation,
biological therapy, and chemotherapy. This patient was most likely infected with HIV
from intravenous drug use.
Alcohol (choice A) and cigarette smoking (choice B) are not associated with Kaposi's
sarcoma.
Pesticide exposure (choice D) may be associated with contact dermatitis, which is a
cutaneous response to an irritant. The typical characteristics are pruritus, erythema,
edema, and vesicles. Kaposi's sarcoma is not associated with pesticide exposure.
Sun-exposure (choice E) is associated with basal-cell carcinoma, squamous cell
carcinoma, and malignant melanoma. Basal-cell carcinoma typically occurs on the face
and is characterized by a papule with central ulceration and a "pearly" border.
Histological evaluation shows nests and clusters of deeply basophilic epithelial cells
with "palisading" nuclei on the periphery. Squamous-cell carcinoma typically occurs on
the face, especially the ears and lower lip, and is characterized by an indurated and
hyperkeratotic plaque with crusting and ulceration. Histological evaluation shows sheets
and nests of neoplastic epidermal cells. Malignant melanoma, which has many clinical
variants, is typically associated with sun exposure in childhood. It can occur almost
anywhere on the skin or on the body and can be melanotic or amelanotic. The typical
lesions occur on the trunk, and extremities, are irregularly pigmented, and have
asymmetric borders. Microscopic evaluation shows neoplastic melanocytes. Kaposi's
sarcoma is not associated with any of these malignancies.
A 69-year-old woman with hypertension and mild anxiety disorder comes to the clinic
because of a 10-day history of lower lumbar pain. She tells you that she is unsure of how
it started and cannot give you a more detailed history. She tells you that she had a
hysterectomy and oophorectomy for unknown reasons 40 years ago. She takes atenolol,
a multivitamin, calcium supplements, and alprazolam. Physical examination is
unremarkable. Plain radiographs of the lumbar spine demonstrate multiple age
indeterminate lumbar compression fractures and hyperlucent vertebra. A pelvic
ultrasound reveals no uterus or ovaries and a normal aorta. The most appropriate next
step in the clinic today is to
A. recommend calcitonin and estrogen replacement therapy
B. recommend calcitonin, estrogen replacement therapy, and ibuprofen
C. recommend estrogen replacement therapy and ibuprofen
D. recommend estrogen replacement therapy only
15
E. send her for x-ray bone densitometry
Explanation:
The correct answer is B. This patient has severe osteoporosis with compression
fractures. Her disease is severe because her ovaries were removed at a young age.
Standard treatment includes estrogen replacement and calcium supplementation along
with moderate exercise. In women who are at least 5 years beyond menopause,
calcitonin slows bone loss, increases spinal bone density, and, according to anecdotal
reports, relieves the pain associated with bone fractures. Calcitonin reduces the risk of
spinal fractures. In addition, the pain of compression fractures, which can be severe,
must be managed with an analgesic such as ibuprofen.
Calcitonin therapy with estrogen replacement (choice A) along with calcium
supplementation is aggressive treatment for osteoporosis. But this patient has known
compression fractures and pain, so analgesia is indicated.
Estrogen replacement with ibuprofen (choice C) along with calcium supplementation
fails to utilize the beneficial effects of calcitonin in this patient population (see above).
Estrogen replacement (choice D) is only a part of a complete treatment program in an
osteoporotic patient with compression fractures. Calcitonin to help increase bone
density and analgesia for the compression fractures are necessary.
X-ray bone densitometry (choice E) measures the density of bone compared to normal
standards. The presence of compression fractures means that there is already severe
osteoporosis. That said, x-ray bone densitometry would be useful to follow the effects of
treatment. It would not, however be the first priority at this clinic visit.
A 19-year-old man is brought to the office by his mother who is concerned about her
son's posture. She states that despite verbal encouragement, physical therapy, and
sports participation, her son slouches forward. She inquires, "Why is his back so round?"
The mother further relates that previously she was informed his back was normal and he
would "grow out of it". She feels that the roundness has actually been progressing. The
patient denies any recent weight loss or gain or change in appetite. He denies any fever,
chills, night sweats, or other constitutional symptoms. He denies any bowel or bladder
dysfunction. There is no history of weakness, extremity paresthesia, or gait
abnormalities. He notes very occasional back pain unrelated to any particular inciting
activity. The pain is non-radiating, does not require analgesic, and is relieved with rest.
There is no history of trauma. He is also bothered by the appearance of his back stating,
"In the summer or while swimming at gym class I usually wear a T-shirt". The past
medical and surgical history is significant only for asthma and an appendectomy in
childhood. Family history is non-contributory. He denies alcohol or tobacco use. Physical
examination reveals a healthy appearing male of average height and slight obesity.
Inspection of the spine region reveals a marked thoracic kyphosis and increased lumbar
lordosis. No focal spinal tenderness or soft tissue changes. Gait is full and symmetric.
Neurovascular examination is normal. X-rays were obtained on 3-foot cassettes of the
thoracolumbosacral spine in AP and lateral projections. The AP projection reveals an 8degree curve in the coronal plane from thoracic vertebral body number 6 to number 11.
The convexity of this curve is to the right. A left convex curve of 7 degrees is noted from
the thoracic body number 12 to lumbar body number 4. The lateral spinal x-ray reveals a
thoracic kyphosis of 60 degrees. The apex of the kyphosis is at thoracic body number 9.
The lateral radiograph also reveals the vertebral body height is 4 mm greater posteriorly
16
than the anterior height at thoracic vertebrae levels 8, 9, and 10. At this time the most
correct statement about his condition is:
A. Inform the family of the benign nature of this condition and that it is essentially
a cosmetic deformity. Emphasize the fact that the patient is having minimal back
pain. The only corrective modality is surgery, but the benefits do not outweigh the
risks. Schedule the patient for clinical and radiographic follow-up appointments every
6 months. Recommend continuation of physical therapy types of exercises at home
as previously performed at physical therapy.
B. Inform the family that this is most likely a benign cosmetic deformity. However,
laboratory studies, including a CBC, ESR, metabolic panel including calcium and
phosphorus levels should be completed. A bone density study should also be
performed to rule out osteoporosis as the cause for the vertebral height differences.
These studies will help rule out infection or a metabolic process as the etiology of
the thoracic kyphosis.
C. This is a benign condition requiring conservative management. Discuss with
the family the importance of a well-structured physical activity program. Write a new
prescription for 8 weeks of physical therapy. The new therapy script should
emphasize strengthening and range of motion exercises for the back and abdominal
regions and a home exercise program.
D. This is a benign condition requiring conservative management. Discuss with
the family the importance of a well-structured physical activity program. Write a new
prescription for 8 weeks of physical therapy. The new therapy script should
emphasize strengthening and range of motion exercises for the back and abdominal
regions and a home exercise program. A hyperextension brace should be prescribed
and worn 23 hours a day. If the thoracic kyphosis does not improve in 6 months,
then an orthopaedic surgery referral is indicated.
E. This is a benign condition requiring conservative management. Discuss with
the family the importance of a well-structured physical activity program. Write a new
prescription for 8 weeks of physical therapy. The new therapy script should
emphasize strengthening and range of motion exercises for the back and abdominal
regions and a home exercise program. In addition, a hyperextension brace should
be prescribed and worn 23 hours a day.
Explanation:
The correct answer is A. This patient has Scheuermann's kyphosis. As described by the
Danish radiologist Holger Scheuermann, it is a round back associated with wedged
thoracic vertebrae. The etiology remains unclear. Theories have included avascular
necrosis of the vertebral bodies, abnormal proteoglycans, and osteoporosis of the
vertebrae. Hamstring tightness and hip flexor tightness have been postulated. It is more
common in males. Patients may complain of occasional aching back pain, but the most
common reason for presentation is the appearance. The parents, and sometimes the
patient, are concerned about the cosmetics of the condition. A complaint of back pain
may be the patient's way of complaining about the social pain of the deformity.
Increased thoracic kyphosis results in a compensatory increased lumbar lordosis.
Normal thoracic kyphosis is less than 45 degrees. A 60-degree kyphosis is moderately
severe. Radiographs of patients with Scheuermann's kyphosis reveal thoracic kyphosis
greater than 45 degrees, with 5 degrees or more of anterior wedging of 3 adjacent
vertebrae. Other x-ray findings include Schmorl's nodes, end plate irregularities, disc
narrowing, scoliosis, and spondylolysis. Physical exam is otherwise unremarkable,
17
except for hyperkyphosis and a sharply angulated bend at the thoracic or thoraciclumbar junction. Adolescents with a fixed or progressive kyphosis require postural
awareness and an exercise program. Bracing is an option for the skeletally immature
and is more successful for curves less than 75 degrees. Teenagers often reject braces.
Painful, severe curves may be corrected surgically. Surgery typically requires an
anterior spinal release followed by a posterior spinal fusion and instrumentation. The
natural history is that most patients will have a worsening of the deformity until skeletal
maturity. Kyphosis less than 60 degrees at skeletal maturity will not predictably have
symptoms in adult life. The differential diagnosis includes postural round back. This
entity has thoracic kyphosis, but there is no vertebra wedging.
This patient does not have an infection, tumor, metabolic disease, or other pathology.
Further tests (choice B) are expensive and unnecessary.
This is a benign condition in a skeletally mature patient. Sending the patient back to
physical therapy (choice C) is unnecessary. Home activity is warranted for all patients,
but will not reverse the deformity of this patient.
Hyperextension braces (choice D) are helpful in the skeletally immature, compliant
patient. This patient is skeletally mature, so bracing will be ineffective.
This is a moderate, relatively non-painful kyphosis. The coronal curves are not
significant. By definition these curves are not scoliosis, since they are less than 10
degrees. Observation is the management with the continuation of a home exercise
program. Spine surgery (choice E) is fraught with potential adverse outcomes, the least
of which can be no improvement of the deformity to paralysis, death, etc.
You are called to see a 45-year-old nursing home resident, who has been there since a
motor vehicle accident that left him paralyzed from the neck down 2 months ago. He
denies any active medical problems prior to his car accident 2 months ago. In the past
month, he has noticed a rash on his back that is occasionally pruritic. He denies any
systemic manifestations associated with the rash. He is confined to his bed and the
nursing staff turns him to his side once per day by propping him back with multiple
pillows. He has notable atrophy of all the extremities. Cutaneous examination reveals
numerous non-folliculocentric inflammatory papules distributed over his posterior trunk.
There is no involvement of the anterior trunk, extremities, face, or oral mucosa. The
most appropriate management of this patient's condition is to
A. prescribe oral minocycline
B. prescribe a topical corticosteroid
C. prescribe a topical tretinoin
D. recommend a topical benzoyl peroxide
E. tell the nurses to rotate the patient on his sides more frequently and keep his
room at a cooler temperature
Explanation:
The correct answer is E. Rotating the patient and keeping the room at a cooler
temperature is correct because miliaria, also known as heat rash, is a common
phenomenon in individuals during prolonged bedrest. It usually involves the posterior
trunk of a bedridden patient. The eccrine sweat duct occlusion is the initial event. The
18
duct ruptures, leaks sweat into the surrounding tissues, and induces an inflammatory
response. The papular and vesicular lesions resemble folliculitis with one major
distinguishing feature. They are not follicular and therefore, do not have a penetrating
hair shaft. The best treatment for miliaria is to keep the areas involved cool and avoid
occlusive clothing.
Oral minocycline (choice A), topical tretinoin (choice C), and topical benzoyl peroxide
(choice D), are incorrect because these are treatments for folliculitis or acneiform
eruptions. Since the inflammatory papules on this patient are described as nonfolliculocentric, folliculitis is not likely the diagnosis.
A topical corticosteroid (choice B) is incorrect because use of corticosteroids topically
may result in miliaria as an adverse reaction.
A 73-year-old obese man comes to the emergency department complaining of a several
day history of a very sore mouth and a 1-day history of "blisters" in his groin, underarms,
and back. He was feeling "fine" before this, but has been generally feeling "worse and
worse" since this began. He tells you he has a history of "reflux" and gout and had an
appendectomy in his youth. He tells you that he takes allopurinol and omeprazole and
that the dose of allopurinol was increased about 5 weeks ago. His temperature is 37.0 C
(98.6 F). The oral cavity has multiple erosions on the hard palate, buccal mucosa, and
gingival surfaces and his breath is foul smelling. The glans penis has similar erosions.
His axillae, inguinal area, and back display a mixture of erosions and flaccid bullae filled
with a yellowish fluid. Lateral pressure on the skin surrounding the bullae cause
sloughing of the epidermis. Biopsy of a representative bulla is obtained and sent for
fresh frozen section, revealing a prominent intraepidermal bulla with prominent
acantholysis. At this time, the most important therapeutic measure is to
A. discontinue allopurinol
B. initiate high-dose corticosteroids
C. initiate methotrexate therapy
D. initiate metronidazole therapy
E. initiate nafcillin therapy
Explanation:
The correct answer is B. This case represents a classic history, physical exam, and
histology of pemphigus vulgaris, a bullous disorder characterized by lesions starting at
the mouth and mucous membranes and then appearing in the intertriginous areas such
as the axilla or groin. The bullae easily rupture leaving erosions; this is especially
prominent in the oral cavity where the initial bullae are almost never observed and
instead only erosions are seen. The bullae are flaccid (as opposed to bullous
pemphigoid, the other classic bullous disorder where they are tense bullae) and are
filled with a clear straw colored fluid (representing serum). The surrounding skin
demonstrated the Nikolsky sign—a sloughing of the skin with lateral pressure—similar
to toxic epidermal necrolysis. There is an autoantibody in these patients that is directed
against desmosomes (cellular connections which hold the cells in the epidermis
together) accounting for the formation of bullae and the histological finding of
19
acantholysis (separation of the cells of the epidermis from each other). Please note that
the history and physical exam alone is sufficient to obtain the diagnosis in this
vignette—knowledge of the histology, while confirming the diagnosis, is not necessary
to answer the question. Until the discovery of corticosteroids, this disease was
universally fatal. High-dose corticosteroids are the cornerstone of therapy and should
be instituted as soon as possible.
Discontinuing the allopurinol (choice A) would be appropriate if the patient had toxic
epidermal necrolysis (TEN). Like pemphigus vulgaris, TEN shows prominent oral
involvement and a positive Nikolsky sign, but bullae are not a prominent feature of the
disease. Both typically severely affect the oral cavity and mucosal surfaces, but TEN
tend to favor the extremities and trunk whereas pemphigus favors the intertriginous
areas. While rarely clinical uncertainty exists between the two, in general the two are
not confused. A biopsy of TEN would show a subepidermal bullae with prominent
epidermal necrosis. Anticonvulsants, allopurinol, and sulfa containing drugs are the
most common cause of TEN.
After pemphigus vulgaris is brought under control with steroids, starting methotrexate
(choice C) or other steroid-sparing agents such as cyclophosphamide, azathioprine, or
cyclosporine is often initiated. These are not appropriate, however, as first-line agents
in the treatment of pemphigus vulgaris.
Initiating metronidazole (choice D) and nafcillin (choice E) would not be useful in
controlling pemphigus vulgaris, but if at a later time the patient were to develop a
bacterial infection they might be essential.
A 19-year-old college student was seen in the emergency department 4 days ago for a
painful lump on her buttocks. She does not remember what "diagnosis" the emergency
department physician gave her, but she was instructed to apply warm compresses to the
lump, keep the area covered with a dry gauze, and take an antibiotic (the name of which
she does not remember). She now comes to you stating that the gauze is getting dirty
with foul-smelling drainage. She is ruining her clothing and has become quite concerned.
She denies any fevers, chills, weight loss, trauma to the area, or similar prior episodes.
There is no contributory past medical, surgical, familial, or social history. Her
temperature is 37.0 C (98.6 F). Her abdomen is soft, nontender, without organomegaly
or palpable masses. There is no inguinal adenopathy. Pelvic examination reveals no
vaginal discharge, cervical motion tenderness, or adnexal masses. There is a 2 cm
mass between the superior area of her gluteal folds. The mass is red, warm, fluctuant,
and very tender. You notice an area of drainage from the mass. Rectal examination
shows good sphincter tone without masses, tenderness, or fluctuance. Stool is guaiacnegative. The most likely diagnosis is
A. an enterocutaneous fistula
B. genital warts
C. an infected Bartholin's cyst
D. an infected pilonidal cyst
E. metastatic rectal cancer
20
Explanation:
The correct answer is D. This woman is suffering from an infected pilonidal cyst. This
cyst, sometimes called a pilonidal sinus, is caused by infection of the hair follicle in the
sacrococcygeal area. It may present as an acute abscess at the sacrococcygeal area
that may rupture spontaneously. The most common presentation is a painful and
fluctuant mass. Initially only cellulites may be present. However, in the chronic stage,
the diagnosis is confirmed by the presence of a sinus opening in the intergluteal fold,
about 5 cm above the anus. Examination may reveal a pit or pits in the midline that
represent infected hair follicles. Treatment of the acute infection includes incision and
drainage of the sinus. All hair must be removed. Long-term treatment may require
extensive excision of all midline pits or sinuses and removal of all hair and debris.
An enterocutaneous fistula (choice A) is an abnormally draining sinus occurring
between the colon and the skin. The foul smelling drainage from this woman's abscess
is pus, not stool. Patients with enterocutaneous fistulas have compromised immune
function and usually have undergone some form of bowel surgery. The fistula is usually
on the anterior abdominal wall.
Genital warts (choice B) may be painful depending on the etiology. They present on the
labia, not the gluteus.
An infected Bartholin's cyst (choice C) occurs when the Bartholin's gland has a duct
(which drains into the distal aspect of the vagina), that becomes obstructed. This
obstruction causes the mucoid secretion to collect and causes dilatation of the cyst.
This collection may get infected. Therapy requires incision and drainage of the gland or
possibly marsupialization of the gland.
This patient is quite young to have rectal cancer (choice E), let alone metastatic rectal
cancer. The combination of no masses on rectal exam, guaiac-negative stool, no family
history, and the location of this mass makes this answer highly unlikely.
A 57-year-old woman comes to the clinic with hoarseness, shortness of breath, cough,
and bilateral otalgia for the past 2 days. She tells you that she has had similar symptoms
in the past several days, but with much less severity. Her past medical history is
significant for moderate arthritis of the knees, ankles, and wrists for the past 30 years.
An extensive evaluation for rheumatoid arthritis has been negative. Vital signs are:
temperature 37 C (98.6 F), blood pressure 120/90 mm Hg, pulse 82/min, and
respirations 9/min. On physical examination, the external ears are tender to touch with
the exception of the lobule of the ear. There is no conductive or neural hearing loss.
There is a saddle nose deformity. The lungs have minimal bibasilar rhonchi. The
abdominal examination reveals a normal size spleen and liver. The ankle and knee joints
are tender but not erythematous. Chest x-ray demonstrates focal tracheal narrowing.
The most appropriate management at this time is
A. an arthrocentesis of the knee joint
B. to order knee and ankle x-rays
C. to prescribe corticosteroids
D. to prescribe nonsteroidal antiinflammatory drugs
E. to schedule a bronchoscopy
21
Explanation:
The correct answer is C. These symptoms are consistent with relapsing polychondritis
(RP). Relapsing polychondritis is a recurrent inflammatory condition of the cartilage
most prominent in the joints, nose, ears, costal cartilage, larynx, and airways. Aortic
aneurysm may also be a complication. The characteristic clinical feature differentiating
RP from other rheumatological conditions is inflammation confined to the cartilage
organs like the ears and nose. Tracheal narrowing is a characteristic radiographic
finding. Corticosteroids are the main form of treatment of RP. High-dose oral
prednisone is used to treat respiratory tract involvement, and intravenous pulse steroids
may be useful for acute airway obstruction.
An arthrocentesis (choice A) is not useful in the management or diagnosis of relapsing
polychondritis (RP). Synovial fluid samples from patients with RP will show a
nonspecific noninflammatory transudate.
Knee and ankle x-rays (choice B) will not contribute to the diagnosis of relapsing
polychondritis (RP). RP manifests as a nondeforming, nonerosive arthritis and x-rays of
the painful joints are often normal.
Nonsteroidal, antiinflammatory drugs (choice D) have been shown to control mild
episodes of inflammation. However, corticosteroids are the mainstay of treatment for
relapsing polychondritis.
Bronchoscopy (choice E) can exacerbate laryngotracheal inflammation and be a cause
of iatrogenic airway obstruction. Hoarseness, signifying laryngotracheal inflammation, is
likely to develop during the course of relapsing polychondritis. Other symptoms of
laryngotracheal involvement include cough, dyspnea, wheezing, and choking.
Involvement of the airway may be localized or diffuse. The larynx and upper trachea are
affected most frequently. Persistent inflammation can destroy the cartilaginous rings
and create luminal collapse. This patient already has tracheal narrowing and a
bronchoscopy may worsen her airway disease.
A frustrated young mother brings in her 2-year-old son with spina bifida for a routine
check. She reports her son's rash, that was present at the last visit, has been worsening.
At the last visit you went through the atopic dermatitis handout with her and
recommended soft, gentle soap and laundry detergent in addition to lukewarm, short
baths, instead of steaming hot, long baths. She tells you that she followed the directions
carefully, and applied emollients on her son's skin after baths everyday. Yet, the rash
continues to worsen. On examination, you note erythematous, lichenified plaques on his
groin areas, upper thighs, and abdominal surface. There is no superficial scaling and no
central clearing of these lesions. There is redness and maceration of the perianal area.
On further questioning, she reveals the perianal rash is from stool incontinence, and
even though she tries to clean him frequently, the area remains red. She proudly adds
that she is very clean and wears latex gloves every time she cleans her son. The most
appropriate management for the rash on the groin and abdomen is
A. oral antibiotic therapy
B. oral antifungal agents
C. topical antifungal therapy and the use of non-latex gloves
D. topical antibiotic therapy and the use of non-latex gloves
22
E. topical steroid cream to the rash twice a day for 2 weeks and the use of nonlatex gloves
Explanation:
The correct answer is E. Using of non-latex gloves is correct because this child has
developed a latex allergy. 3-10% of spina bifida patients eventually develop
sensitization to latex, hypothesized to be secondary to the increased exposure to
surgical procedures in their early life. The history of using latex gloves to clean him
regularly for bowel movements as well as erythematous plaques in groin and abdominal
areas should raise the suspicion for latex allergy. Half of these patients will have
concomitant fruit allergies to banana, avocado, kiwi, chestnut, and passion fruit. The
best treatment is to avoid latex material and treat flares with strong topical steroids.
Oral antibiotics (choice A) and topical antibiotics (choice D) are incorrect, because while
these two treatment options may decrease the chance of superinfection, they will not
treat the primary lesions.
Oral antifungal agents(choice B) and topical antifungal (choice C) are incorrect because
the lesions are not classic for tinea. Tinea generally presents as erythematous bordered
plaques with central clearance and superficial scaling. Occasionally, tinea cruris may
not have scaling secondary to the moist environment, but the lesions on his thighs and
abdomen would have the classic scales.
An 18-year-old boy with Hodgkin's lymphoma is admitted to the hospital for his third
course of chemotherapy which includes prednisone. On his 6th hospital day, you are
called by the nurse to assess a new rash which has slowly progressed to cover the
entire anterior and posterior trunk, as well as his arms and part of the face. The nurse is
concerned about a possible allergic reaction, secondary to the multiple medications this
patient is taking. On examination, you note numerous, small, pinpoint pustules and
inflammatory papules of the same stage scattered mostly over his trunk and proximal
arms. There are some pustules of the same stage over his forehead as well. The patient
denies any significant history of acne prior to the initiation of chemotherapy. At this time
the most correct statement about his condition is:
A. This is acne vulgaris and should be treated with oral minocycline and topical
benzoyl peroxide
B. This is a drug eruption and you should stop all the medications and reinitiate
one every week
C. This is a drug eruption secondary to his chemotherapy and topical
corticosteroid will alleviate the problem
D. This is miliaria secondary to the occlusion effect by his hospital gown
E. This is steroid acne and topical tretinoin cream may help
Explanation:
The correct answer is E. Acneiform eruptions is correct because it is characterized by
papules and pustules resembling acne lesions, not necessarily confined to the usual
sites of acne vulgaris. The eruptions are distinguished by their sudden onset. Oral
medications such as iodides, bromides, testosterone, cyclosporine, antiepileptic
medications, lithium, and systemic corticosteroids are common agents that can lead to
acneiform eruption. When medium or high doses of corticosteroids are taken for as
23
short a time as 3-5 days, a distinctive eruption may occur, known as steroid acne. It is a
sudden out-cropping of inflamed papules, most numerous on the upper trunk and arms,
but also seen on the face. The lesions typically present as papules rather than
comedones. Tretinoin cream applied once or twice daily may clear the lesions within 13 months, despite the continuation of high doses of corticosteroid.
Acne vulgaris (choice A) is incorrect because of the atypical distribution (proximal
arms), and the sudden onset of numerous lesions.
An allergic drug eruption (choice B) is incorrect because typical drug eruptions do not
present with pustules. His medication should not be terminated.
Typical drug eruptions do not present with pustules and topical steroids (choice C),
especially of the fluorinated types, or when applied under occlusion, may also induce an
acneiform eruption and worsen this patient's skin problem
Miliaria (choice D) is incorrect. This is retention of sweat as a result of the occlusion of
eccrine sweat ducts and pores, producing an eruption that is common in patients with
prolonged bedrest. These patients typically present with numerous folliculocentric
inflammatory papules on the posterior trunk, resulting from extended periods of being
on their back. The face, central chest, and arms are typically not occluded in bedrest
patients.
A 28-year-old psychiatric patient jumps out of his bedroom window (2 stories up),
landing on his feet. On arrival to the emergency department, he is conscious with no
evidence of cranial injury. His blood pressure is 130/80 mm Hg and pulse is 80/min. His
chest and abdomen examination are unremarkable, except for tenderness on palpation
of bony prominences. Pelvic examination shows no compressive tenderness.
Examination of his extremities shows bilateral ankle tenderness. Initial x-rays reveal
bilateral calcaneal fractures. This patient is at increased risk for
A. a femoral shaft fracture
B. a knee dislocation
C. lumbar fractures
D. a pneumothorax
E. a rib fracture
Explanation:
The correct answer is C. Bones more prone to fracture on landing on feet are the
calcaneum and spine. Fractures of the calcaneum and spinal compression fractures
need to be excluded in jumpers. Also, posterior hip dislocation and acetabular fractures
can result due to forceful movement of the femur in a posterior direction.
Femoral shaft fractures (choice A) happen due to direct injury and can result in massive
amounts of bleeding without any external evidence. Patients with femoral shaft
fractures are usually hemodynamically unstable due to the massive amount of blood
loss.
The knee is a stable joint and dislocation can happen due to direct trauma or due to
trauma involving fixation of thigh or leg. A knee dislocation (choice B) is unlikely from
landing on feet.
24
A pneumothorax (choice D) results from direct injury to the chest, however this patient
landed on his feet.
Rib fractures (choice E) result from direct injury to the chest. In young adults, ribs are
not osteoporotic and hence require significant trauma to fracture.
You are called by the surgical team to evaluate a new rash on a 55-year-old man who
underwent a colectomy for colon carcinoma 10 days prior. He recalls complaining of
pressure-induced pain on his buttocks prior to surgery, and the internal medicine nursing
staff placed "something" on his back to alleviate the pain. After the surgery, he was
transferred to surgical team care. He complains of severe itching and occasional pain on
his buttocks. You remove the dressing that is overlying the sacral area and find an
erythematous, well-demarcated patch matching the size and shape of the dressing. The
surrounding skin is normal and there is no lymphadenopathy. The surgical team is
concerned that this is an infectious process. He is afebrile and has been receiving
multiple intravenous antibiotics since the surgery. The most appropriate management is
to
A. add an intravenous anti-streptococcal antibiotic to treat erysipelas
B. perform a punch biopsy to rule out a malignancy
C. start intravenous immunoglobulin therapy to treat toxic epidermal necrolysis
D. stop all of his antibiotics and resume one at a time because this is most likely
due to a drug eruption
E. recommend a topical corticosteroid to the area twice a day to treat contact
dermatitis
Explanation:
The correct answer is E. Topical steroids are the correct treatment for the irritant
contact dermatitis to DuoDerm dressing on this patient. The biggest hint in this vignette
is that the rash matches the size and shape of the dressing that was removed from the
patient. Often times, irritant dermatitis produces severe pruritus without other
symptoms.
Adding IV antibiotics to treat erysipelas (choice A) is incorrect because erysipelas is an
acute, inflammatory form of cellulitis that has prominent lymphatic involvement
(streaking).
Performing a punch biopsy (choice B) is incorrect, again, because the clinical exam is
classic for contact dermatitis. Cutaneous metastasis can present in various shapes and
forms, but is much less common.
Starting immunoglobulin therapy (choice C) is incorrect because this patient has no
fever or oral or conjunctival mucosal involvement to suggest toxic epidermal necrolysis
secondary to medications.
Stopping all of antibiotics (choice D) is incorrect, because the typical antibiotic-induced
drug eruption is not limited to one local area with well-demarcated borders.
25
A previously healthy 26-year-old office manager comes to the clinic complaining of a
widespread rash present for several days. She states she had a large, oval, scaly area
on her abdomen arise 2 weeks ago. She treated it with an over-the-counter topical
antifungal cream that seemed to be helping, although she says that original rash is still
somewhat there. Two days ago a multitude of smaller scaly oval plaques appeared over
her "entire body." She states the eruption is asymptomatic except for the appearance.
She lives with her 2 children, ages 3 and 5, and their dog. She was in a monogamous
sexual relationship with a man, but that ended 8 months ago and she has not had any
sexual contact since that time. She is healthy and does not take any medications.
Physical examination shows an oval plaque measuring about 4 cm by 7 cm with a
prominent collarette of scale on her left mid abdomen. Many smaller but similar
appearing oval plaques measuring about 1-2 cm are present, scattered on most of her
torso. On the back these are arranged in a "christmas tree" pattern. She is worried about
the rash's appearance and would like treatment as her family is coming to visit her in 1
week. The next most appropriate course of action is to
A. obtain a RPR or VDRL and while awaiting the results administer 2.4 million
units of penicillin intramuscularly
B. prescribe ketoconazole lotion, telling the patient to apply it each night to her
torso for 1 week and then weekly as prophylaxis
C. prescribe oral fluconazole and tell her to bathe her dog in antifungal shampoo
D. prescribe a tapering course of prednisone, but warn her that as her condition
is thought to be inherited it may come back at some point in her life.
E. reassure her about the benign and self-limited nature of her condition and that
it will likely resolve in several weeks.
Explanation:
The correct answer is E. This case describes a classic history and examination for
pityriasis rosea. This condition usually affects children and young adults, typically in the
spring or fall, seems to occur in epidemics, and while the etiological agent is still
unknown, is almost assuredly a viral infection. The eruption is characterized by the
appearance of a "herald patch," which is a large and scaly, typically oval plaque on the
torso. Occasionally the patients will describe a mild upper respiratory infection or mild
flu-like symptoms around the time the herald patch arises. Several weeks later
"daughter lesions" appear on the torso which are like smaller versions of the herald
patch. On the back these typically follow the lines of skin cleavage and hence resemble
a "Christmas tree." The disease is self-limited in almost all cases and will
spontaneously resolve in several weeks after the appearance of the daughter lesions.
No treatment is known to hasten recovery.
The eruption of pityriasis rosea and the rash of secondary syphilis can be very similar.
Thus checking the patient for syphilis (choice A) is certainly reasonable if the patient
has had unprotected sexual contact within the past 6 months. If patient adherence to
follow-up is a concern, immediate initiation of treatment is also recommended. In this
situation, however, it is unlikely the patient has syphilis. Secondary syphilis does not
typically have a "herald patch" and, unless one is immunocompromised, secondary
syphilis occurs within 6 months of infection (usually within the first 3 months). The
patient in this vignette states her last sexual contact was over 8 months ago.
Ketoconazole lotion (choice B) is effective treatment and prophylaxis for tinea
26
versicolor, also called pityriasis versicolor. It is a superficial yeast infection of the skin,
almost always presenting during the summer months. It is characterized by a multitude
of small scaly patches on the upper torso and occasionally proximal arms. Confirming
the diagnosis can best be done by examining a KOH preparation looking for yeast.
Tinea versicolor is not associated with a herald patch. Having the initial lesion on the
abdomen and for it to initially present in November would also be extremely usual.
Prescribing a 1-week course of oral fluconazole (choice C) may be a reasonable course
of action if the patient had widespread tinea corporis (fungal infection of the skin). Tinea
corporis typically presents as a single roughly circular plaque with prominent scale. This
scale is usually accentuated at the periphery. Left untreated it usually slowly grows in
diameter. Adults typically obtain tinea corporis from children or pets, thus asking that
the family dog be washed in antifungal shampoo would be sensible if the pet had any
evidence of fungal infection. Fungal infection could be confirmed by examining a KOH
preparation looking for hyphea. Widespread fungal infections without any known
predisposing factor such as diabetes mellitus can be a clue to immunocompromised
states and thus, depending on the situation, testing for HIV may be indicated.
Occasionally pityriasis rosea can be misdiagnosed as psoriasis. Psoriasis is typically
comprised of erythematous plaques on the extremities and torso with abundant silvery
scale. There is a genetic component to the disease and lifelong recurrences are the
norm. In this vignette if you prescribe oral steroids (choice D) you will likely exacerbate
the eruption of pityriasis rosea and thus it is clearly contraindicated. In addition, if you
suspect the patient has psoriasis, oral steroids may result in dramatic improvement of
the rash but almost without exception after the steroids are stopped a "rebound" of the
psoriasis occurs and it comes back far more aggressively. It may even trigger pustular
psoriasis which can be life threatening.
A 72-year-old man comes to the office because his wife has been nagging him about
"ugly growths" on his hands, forehead, and ears. He says that he began to notice them a
few months ago, but lately they have become even more "unsightly". Now that he is
retired, he spends most of his free time on his boat, which he calls his "baby". He says
that he knows that he should use sunscreen, but "who remembers." He is very
suntanned. Physical examination shows multiple scaly, erythematous macules and
papules, ranging from 0.1-cm to 0.8-cm. A couple of the lesions on his ears and
forehead have a curved horn-like appearance. The most likely diagnosis is multiple
A. actinic keratoses
B. basal cell carcinomas
C. keratoacanthomas
D. seborrheic keratoses
E. squamous cell carcinomas
Explanation:
The correct answer is A. This patient most likely has multiple actinic keratoses (AK),
which are common lesions seen on sun-exposed skin. AKs are a cutaneous dysplasia
of epidermis that may undergo malignant transformation. They typically occur in middle-
27
aged and elderly adults. AKs can be macules or papules, and often have a
hyperkeratotic, adherent scale. The "cutaneous horn" is caused by an abnormal growth
pattern of the cells within the AK, that resembles an animal horn. The treatment
includes cryosurgery, surgical excision, pathologic evaluation, and topical therapy.
Basal cell carcinomas (choice B) are typically characterized by a "pearly papule", or a
papule with central ulceration and a scaly, "pearly", rolled border.
A keratoacanthoma (choice C) is a rapidly growing nodule that has keratinous debris in
the center. It is related to squamous-cell carcinoma.
Seborrheic keratoses (choice D) are benign lesions that can range from tan to dark
brown, and usually have a waxy "stuck-on" appearance. They are typically found on the
back and face.
Squamous cell carcinomas (choice E) are malignant neoplasms that usually occur on
sun-exposed areas of the body, especially on the ears and lower lip. They often appear
as a hyperkeratotic, crusted, and ulcerated plaques.
A 45-year-old woman is in the hospital following a major motor vehicle accident. She
sustained fractures of her left femur and left tibia. She feels well after her open reduction
and internal fixation of her fractures. On her third hospital day, she mentions that she
has pain in her jaw. Since her hospitalization, she has had difficulty opening her mouth
completely and when she does, she often hears a "popping" sound. Physical
examination reveals moderate pain with palpation of the mastication muscles bilaterally.
She is able to slowly open her mouth completely. There are no facial lacerations or
bruising. At the time of admission, the patient had a CT of the head that was normal. The
most appropriate step at this time is
A. an arthrography of the temporomandibular joints
B. a CT of the temporomandibular joints
C. an MRI of the temporomandibular joints
D. to recommend a soft diet to limit chewing
E. to repeat the CT of the head
Explanation:
The correct answer is C. The signs and symptoms of jaw pain, limitation of mouth
excursion, and "popping" during mouth opening are most consistent with
temporomandibular joint (TMJ) derangement or dislocation. Although most cases are
idiopathic, there are predisposing factors such as trauma, dentition problems, joint
conditions, and hyperactivity of the muscles of mastication as with bruxism. An MRI is
the best diagnostic tool for evaluating all the structures of the TMJ. The disk of the TMJ
most commonly dislocates anteriorly.
An arthrography (choice A) is accurate in the diagnosis of internal derangements of the
temporomandibular joint (TMJ). It is not frequently performed anymore with the advent
of MRI. Arthrography is an invasive procedure associated with complications and
patient discomfort that are readily avoided with MRI.
A CT (choice B) is not reliable in the diagnosis of temporomandibular joint (TMJ)
dislocation because the disk is inconsistently visualized. It is a good imaging modality to
28
evaluate the bony structures of the jaw, but it is limited in its resolution of the disk,
articular cartilage, and tendino-ligamentous insertions.
Recommending a soft diet to limit chewing (choice D) may help the patient avoid pain. It
does not, however, address the underlying issue of a probable temporomandibular joint
(TMJ) dislocation. An MRI of the TMJ is the most appropriate way to make this
diagnosis.
Repeating the CT of the head (choice E) is not indicated in this patient. Her symptoms
are limited to the temporomandibular joints (TMJ) and there is no reason to suspect an
intracranial process as an etiology. A CT of the head does not include the TMJ in the
scan. An MRI of the TMJ is the most appropriate way to make the diagnosis of internal
derangements.
A 12-year-old boy with asthma is brought to the emergency department by his mother
because of intermittent right hip pain for the last 2 weeks. The pain is non-radiating and
worse with activity. It has now become more constant, worse with weight bearing, and
over-the-counter analgesics only give minimal relief. There is no history of night pain and
he denies any recent trauma, weight change, or any constitutional symptoms such as
fever, chills, or night sweats. He tells you that he went to his pediatrician's office 10 days
ago for the same hip pain and he was told that his physical examination and laboratory
studies, including a complete blood count and erythrocyte sedimentation rate, were
unremarkable. The pediatrician's diagnosis was a "pulled muscle or tendon" in the right
hip region and he was advised to rest. Now in the hospital, his physical examination
shows an obese patient, a limping gait, a leg length discrepancy of 0.5 cm, and intact
motor-sensory examination of lower extremities bilateral. The right hip region has intact
skin and no focal tenderness to palpation. Passive range of motion of the right hip is
decreased on internal rotation. When the hip is flexed, the thigh externally rotates. The
most appropriate next step in management is to
A. admit him to the hospital for intravenous antibiotic therapy, non-weight bearing
right lower extremity, and obtain an orthopaedic consultation
B. recommend immediate crutch walking and non-weight bearing of right lower
extremity and obtain pelvic x-rays
C. repeat complete blood count and erythrocyte sedimentation rate; if the results
are normal then send him home with 1 week follow-up in the office
D. send him home with a prescription for a 0.5 cm shoe insert and physical
therapy
E. send him home with a 10-day course of oral antibiotics with a 2-week follow-up
in the office
Explanation:
The correct answer is B. A skeletally immature and obese 12-year-old patient with this
history of hip pain is most likely to have a slipped capital femoral epiphysis (SCFE).
This growth plate condition does not occur after skeletal maturity, and for males,
maturity is on average at 16 years of age. The obligatory external hip rotation with hip
flexion is a very common finding in a slipped epiphysis. Immediate non-weight bearing
protects the femoral epiphysis from further injury and potential osteonecrosis. Common
x-ray findings are consistent with a slippage of the femoral epiphysis. The slippage has
been compared to a scoop of ice cream that has partially slipped from the cone. A
29
double density sign and a break in Klein's line are two x-ray findings. Treatment
requires internal fixation of the epiphysis with long screws. The risks of delayed or nontreatment are further slippage and osteonecrosis.
An orthopaedic consult (choice A) will result in discontinuance of the antibiotics and
ordering pelvic x-rays. Non-weight bearing is correct and protective, but antibiotics are
not indicated because this is not an infectious process.
This patient does not have transient synovitis or any other infectious or inflammatory
processes.Therefore, repeating a complete blood count and erythrocyte sedimentation
rate, and sending him home if the results are normal with a one week follow up is not
necessary(choice C) is incorrect. Again, a delay in diagnosis will result in significant
morbidity.
Leg length discrepancy is very common, and less than 2 cm does not typically require
treatment. Patients are very often misdiagnosed as having muscle strains or tendon
injuries. This delay in diagnosis results in significant morbidity. Sending him home with
a prescription for a 0.5 cm shoe insert and physical therapy (choice D) does not
address the problem.
This patient does not have an infection. Therefore, antibiotics and discharge to home
(choice E) will only delay the correct diagnosis.
A 34-year-old man comes to the office because of a 5-month history of a left-sided
headache and jaw pain. The headache usually feels dull, achy and radiates to his ear.
He says that it almost feels as if his "jaw is off-centered", and it often "clicks" when he
eats or chews gums. There is a constant feeling of jaw stiffness, and it feels as if it is
"sticking" when he tries to open his mouth. He denies any fever, shortness of breath,
changes in weight or vision, or any other arthralgias, and states that he is otherwise in
good health. His temperature is 37 C (98.6 F), blood pressure is 115/80 mm Hg, pulse is
65/min, and respirations are 12/min. Laboratory studies show:
The most appropriate next step to establish a diagnosis is to
A. administer corticosteroids immediately
B. auscultate the left carotid artery
C. palpate the area just in front of the left tragus
D. palpate the left posterior auricular region
E. palpate the left temporal artery
Explanation:
The correct answer is C. This patient most likely has a temporomandibular joint (TMJ)
disorder, which is a very common disorder that can usually be detected by palpating the
area just in front of the tragus. Joint clicking may be found when the patient opens and
30
closes his mouth. Symptoms of TMJ include orofacial pain, a noisy joint, and restricted
jaw function.
Corticosteroids (choice A) are necessary in cases of temporal arteritis, when the patient
complains of visual changes. However, this patient is a young man who does not
complain of a headache over the temporal artery or a change in vision. He is afebrile,
has a normal hemoglobin, hematocrit, and erythrocyte sedimentation rate, and does not
have any systemic symptoms. It is unlikely that he has temporal arteritis.
If a transient ischemic attack is suspected, auscultation of the left carotid artery (choice
B), is appropriate to detect the presence of a carotid bruit. However, this young patient
is describing the 3 main symptoms of TMJ which include orofacial pain, a noisy joint,
and restricted jaw function. It is unlikely that he has carotid artery disease.
This patient most likely has a temporomandibular joint (TMJ) disorder, which is a very
common disorder that can usually be detected by palpating the area just in front of the
tragus, not the left posterior auricular region (choice D). Here is a practical explanation:
put your finger in front of your ear, and open and close your mouth. You can feel the
movement of your jaw. Now, put your finger behind your ear, in the posterior auricular
region, and open and close your mouth. You cannot really feel your jaw move.
If temporal arteritis was suspected, particular attention should be given to the left
temporal artery (choice E). However, this patient is a young man who does not
complain of a headache over the temporal artery or a change in vision, is afebrile, has a
normal hemoglobin, hematocrit, erythrocyte sedimentation rate, and does not have any
systemic symptoms. Palpation of the temporal artery is appropriate during the
examination to rule out temporal arteritis, but particular attention should be given to the
area just in front of the left tragus, to detect TMJ.
A 21-year-old college football player comes to the university student health center clinic
because of pain and swelling of his right knee for the past 2 weeks. He says the pain
started after he was tackled during football practice 2 weeks ago. Initially, the pain was
severe and he thought he heard a “pop” right after he was tackled. He was not able to
walk immediately after the injury, but he has been walking normally for the past week.
An x-ray of the right knee on the day of the injury was normal. On physical examination,
there is a small effusion in the right knee. There is no erythema or focal areas of
tenderness. He has full range of motion in the right knee and has a positive anterior
drawers test. The most appropriate next step is to
A. order a CT of the knee
B. order an MRI of the knee
C. perform arthrocentesis
D. prepare him for arthroscopy
E. repeat x-ray of the knee
Explanation:
The correct answer is B. The presentation of knee pain following trauma with an
associated “pop” and the immediate inability to ambulate is highly suspicious for a
rupture of the anterior cruciate ligament (ACL). The ACL is frequently injured in contact
31
sports such as football. Patients will also often complain of episodes of instability or
giving-way after the initial symptoms of pain and swelling resolve. The anterior drawer
test involves firmly pulling the lower leg above the calf forward to test for laxity. If the
ACL is ruptured, the lower leg is able to move anterior to the femur. The most
appropriate study to diagnose ACL injury is an MRI. An MRI is the best imaging
modality to study the soft tissues, ligaments, and menisci of the knee. When torn, the
ACL is most often simply not visualized. Sometimes, the actual disruption is seen. A
partial tear of the ACL will demonstrate an abnormally high signal in the ligament. An
MRI can also detect associated injuries such as a medial meniscus tear.
A CT of the knee (choice A) is not a useful study for this patient. A CT is very good in
the evaluation of the bones of the knee and may demonstrate fractures not detected on
the x-ray. It is not a good diagnostic tool in the evaluation of the ligaments, tendons,
and menisci of the knee or other joints.
Arthrocentesis (choice C) is a procedure used to obtain synovial fluid for further
analysis. This procedure is used to diagnose infections and crystalline disease of the
joints. Although the patient has a knee effusion, it is not necessary to analyze the
synovial fluid in his particular presentation. The most likely etiology of the effusion is an
inflammatory reaction related to his primary diagnosis of an anterior cruciate ligament
injury.
Arthroscopy (choice D) is necessary to repair the ruptured anterior cruciate ligament
(ACL). However, it is not the most appropriate procedure to order at this time. It is
critical to obtain an MRI of the right knee, to not only confirm a suspected ACL tear, but
to also evaluate for other concomitant injuries to the other ligaments and menisci of the
knee. Traumatic injury to the ACL is often associated with tears in the medial meniscus
and the medial collateral ligament. An MRI of the knee will provide a road map for
orthopedic surgeons during the therapeutic arthroscopic procedure.
A repeat x-ray of the knee (choice E) is not a useful study for this patient. The initial xray was normal, and the patient's symptoms have not changed since this study. A plain
x-ray of the knee is good for evaluating the bones and for an effusion. It is useful initially
for demonstrating fractures that are highly suggestive of internal derangements in the
knee. A Segond fracture is a tiny avulsion off the posterior lateral tibial joint line and is
almost always associated with a tear of the anterior cruciate ligament. An x-ray is not a
good diagnostic tool in the evaluation of the ligaments, tendons, and menisci of the
knee or other joints.
A 51-year-old woman comes to the office because of a "lump" in her left axilla. She says
that she noticed it 3 months ago in the shower and it has been slowly increasing in size.
She has not seen a physician in ages because her mother died from ovarian cancer at
age 42 and she is angry at "all physicians" because they did not find it before it was too
late. She is generally very healthy, exercises regularly, eats a low-fat diet, does not
smoke cigarettes, lives alone with her dog, and has not traveled recently. She is allergic
to flowers and cats. She denies fever, night sweats, or fatigue. Physical examination
shows a 2.0 cm fixed round lesion in her axilla. The remainder of the examination,
including a clinical breast examination and pelvic examination, are normal. A complete
blood count is unremarkable. The most appropriate next step is to
A. obtain cultures for Epstein-Barr virus and Cytomegalovirus
32
B. order a complete biochemical profile and chest x-ray
C. reassure her and have her return in 2 weeks for reexamination
D. schedule a mammography
E. send her to a surgeon for a lymph node biopsy
Explanation:
The correct answer is D. In general, a patient with axillary adenopathy, without fever,
weight loss, fatigue, and night sweats should be evaluated for cellulitis, cat-scratch
disease, sporotrichosis, and breast cancer. Since the first 3 disorders can basically be
ruled out with a complete history and physical examination, a mammography is needed
to evaluate for breast cancer.
Epstein-Barr virus and Cytomegalovirus (choice A) are typically associated with cervical
adenopathy, not a single, fixed axillary lymph node.
A biochemical profile and chest x-ray (choice B) are not indicated at this time in this
patient with a single fixed axillary lymph node, since she has a family history of cancer
and has not been to a doctor in "ages" and is probably due for a mammogram.
Since this lymph node has been present for months, it is inappropriate to reassure her
and have her return in 2 weeks for reexamination (choice C) because it is unlikely to
decrease in size spontaneously by then. She requires evaluation at this time.
A lymph node biopsy (choice E) would be the appropriate step if this was a
supraclavicular lymph node. However, the evaluation of an axillary lymph node in a
woman typically includes a mammogram with a biopsy if a lesion is found. A lymph
node biopsy should be performed after a breast lesion is ruled out.
A 61-year-old woman is admitted to the hospital for fatigue and pain in her hands and
arms. She has a long history of rheumatoid arthritis and has had multiple surgical
procedures to correct her upper extremity deformities. Over the past few days, she has
had an exacerbation of her disease such that she has been unable to perform her
activities of daily living. She complains of profound fatigue, fever, and a recent 10 pound
weight loss in one month. She has no tobacco or ethanol history. Her medications
currently include gold, methotrexate, and thiazide daily. Her temperature is 38.0 C
(99.4F), blood pressure is 130/55 mm Hg, pulse is 92/min, and respirations are 14/min.
She has bilateral ulnar deviation and a number of 2-3 cm subcutaneous nodules across
her fingers and elbows. Her lungs are clear, but her spleen is palpable in the left upper
quadrant. Chest radiograph reveals multiple 1 cm pulmonary nodules in both lung fields.
The most appropriate next step in management is to
A. determine rheumatoid factor level
B. increase the dose of methotrexate
C. initiate cyclophosphamide therapy
D. order radiograph of the hands
E. schedule an open lung biopsy
33
Explanation:
The correct answer is C. The patient has severe rheumatoid arthritis. She has all of the
classical findings, including morning stiffness, polyarticular involvement, and
rheumatoid nodules. The etiology of RA is unknown but it is a progressive, destructive
disease. The disease is seen in women at least twice as often as in men. It has a
prevalence of nearly 10% worldwide in persons over the age of 65. Her radiographic
findings indicate rheumatoid nodules (Caplan syndrome) and her splenomegaly,
coupled with likely neutropenia, is Felty syndrome seen with severe RA. For these
patients, once they have failed traditional management strategies, immunosuppressive
agents, such as cyclophosphamide or azathioprine, are indicated.
A rheumatoid factor level (choice A) will not be of any value in managing this patient.
Although RF levels often correlate with disease severity, they have no prognostic
significance nor do they reflect efficacy of therapy.
Increasing the dose of methotrexate (choice B) will likely have minimal effect on the
course of this patient's disease. Given her recent exacerbation and evidence on
physical examination of severe disease, a more aggressive approach to treatment is
warranted for this woman.
A radiograph of the hands (choice D) is not needed to evaluate this patient's findings.
She has classic ulnar deviation and rheumatoid nodules. A radiograph will only disclose
additional likely bony deficits secondary to the destruction of bone and cartilage from
her RA process.
Although the nodules, coupled with the fatigue and weight loss could suggest cancer,
an open lung biopsy (choice E) is not the appropriate diagnostic step to take for
evaluation of such symptoms. A CT scan of the thorax would precede such an invasive
option. However, this patient has no risk factors for lung cancer and has nodules in
association with severe RA, making Caplan syndrome the most likely diagnosis.
A 10-year-old African American boy is brought into the office by his mother because of
concerns over "white spots" on his skin that are increasing in size and number. His
mother reports the first spot began 2-3 years ago on his right index finger, which was
stable until recently when it increased in size. Of more concern to the mother was the
increase in number of similar lesions of various size around his mouth, anus, and penile
tip. Further questioning reveals a family history of diabetes and thyroid disease. Full skin
examination shows depigmented, flat patches that are well demarcated with no scales or
surrounding erythema located at the perioral, perianal, tip of penis, and bilateral knees
as well as right index and middle fingers. Management of this patient should include
A. complete blood count and chemotherapy
B. ketoconazole shampoo 3 times per week and oral ketoconazole for 2 weeks
C. sunscreen and sun avoidance
D. topical antifungal cream
E. topical corticosteroids and phototherapy
Explanation:
The correct answer is E. Topical steroid and phototherapy are the two main therapeutic
34
modalities in treating vitiligo. Vitiligo is an acquired disorder of depigmentation
characterized by loss of melanocytes from the epidermis, the mucous membranes, and
other tissues. A vitiliginous lesion usually presents as either a hypopigmented or snowwhite, well circumscribed macule or patch. In most cases, a single lesion is initially
noticed and new lesions can develop either immediately or over an extended period of
time. Lesions progress in size and number. The most common sites are the face,
hands, legs, glans of the penis, and the perianal region. Thyroid diseases (both hypoand hyperthyroidism as well as Graves disease), diabetes mellitus, pernicious anemia,
Addison disease, multiglandular insufficiency syndrome, and alopecia areata have been
associated with vitiligo.
Complete blood count and chemotherapy (choice A) is appropriate if you suspect
cutaneous T cell lymphoma. It is a chronic, frequently fatal disease of helper T cells
commonly seen in patients in the fifth to seventh decades of life. Clinically, the lesions
can present as erythematous, pruritic, brown-to-purple indurated plaques with central
clearing or multiple, round, dome-shaped tumors. Pruritus is usually severe and
excoriations are prominent.
Topical and oral ketoconazole (choice B) is incorrect because these are the mainstay of
therapy in tinea versicolor. It is an extremely common superficial fungal disorder
characterized by multiple scaling, oval, macular, and patchy lesions usually distributed
over the upper portions of the trunk, proximal arms, and occasionally the face. The
lesions are hypopigmented or hyperpigmented, caused by Pityrosporum orbiculare.
Generally asymptomatic, the lesions become lighter than the surrounding skin in the
summer and relatively darker during winter, hence the name tinea versicolor.
Sunscreen and sun avoidance (choice C) is incorrect because these are treatments for
pityriasis alba. This is a common disorder in children, characterized by discrete
asymptomatic hypopigmented patches on the face, neck, upper trunk, and proximal
extremities. Most cases appear following sun exposure and result from a disturbance in
pigmentation of the affected areas.
Topical antifungal cream (choice D) is the treatment for tinea corporis (ringworm), which
is characterized by one or more annular, sharply circumscribed scaly patches with a
clear center and scaly vesicular, papular, or pustular border. The disorder is most
commonly seen in children and in individuals in warm, humid climates.
A worried mother brings in her 7-year-old son who recently returned from summer camp
with a new "itchy" rash on and around his umbilicus. There is a similar rash on his left
wrist, under his brand new metal watch that he received for his seventh birthday before
he left for the camp. His temperature is 37.0 C (98.6 F). He opens his jeans to show you
a well-demarcated, erythematous, circular plaque with numerous small vesicles at the
periumbilical area. The surrounding skin is normal without xerosis. There is a circular,
well demarcated erythematous plaque with similar vesicles on his left wrist. Oral and
conjunctival mucosa, as well as the remainder of the cutaneous examination is
unremarkable. The boy appears happy but the mother is very concerned. At this time the
most appropriate next step is to
A. explain that this is a drug allergy and question her further about new
medications
B. explain that this is a nickel allergy and that he should avoid metals in clothing
and jewelry that contain mixtures of nickel
35
C. reassure her and tell her Molluscum contagiosum often resolves without
treatment in children
D. tell her that this is childhood eczema and that he will most likely require lifelong treatment
E. tell her that this is a viral exanthem which will resolve spontaneously
Explanation:
The correct answer is B. Nickel is the most common cause of allergic contact
dermatitis. The critical factor is the amount of nickel released from metals in intimate
contact with the skin. Sweat, containing sodium chloride, may exacerbate the dermatitis
in persons who sweat profusely. The factors in this setting, which include the jeans
(metal buttons), his watch with a metal base, and the hot summer season, are important
clues to nickel contact dermatitis. Therapy includes avoidance of metal to skin (wear
pants with elastic bands) and the application of topical corticosteroids to affected areas.
Drug allergy (choice A) is incorrect because usually drug reactions result in generalized
morbilliform, erythematous papules that begin on trunk and extend peripherally. No
culprit medication was given in the history and this patient has localized, welldemarcated lesions that are inconsistent with drug eruption.
Molluscum contagiosum (choice C) is incorrect because poxvirus infections tend to be
grouped, flesh-colored, or pearly papules a few millimeters in diameter. As they
progress, they develop a distinct central depression (umbilication) and a white, curd-like
core may easily be expressed. The lesions are generally asymptomatic.
Eczema or atopic dermatitis (choice D) is incorrect because atopic dermatitis patients
have generalized xerosis and tend to worsen in the winter months. Sites of predilection
in children are antecubital and popliteal fossa, face, and neck. Other findings in these
patients are keratosis pilaris, accentuated palmar creases, lichenification, cataracts,
and "allergic shiners" (infraorbital discoloration).
A viral exanthem (choice E) is incorrect because viral exanthems tend to be generalized
and, depending on the virus, can have associated systemic findings such as upper
respiratory symptoms, fever, cough, and oral mucosa involvement.
A 43-year-old female former nurse comes to the clinic complaining of a rash around a
healing laceration on her left dorsal forearm. She reports that she cut her forearm with a
knife while removing it from her dishwasher 5 days ago. The cut was not deep and she
did not seek medical attention. She has been cleaning it with hydrogen peroxide and
applying neomycin ointment, followed by wrapping it in a bandage twice a day.
Yesterday evening during the dressing change she noted some pruritus and erythema in
the area of the wound; this morning she was alarmed to find the area extremely itchy
and with a "horrible rash." She denies fevers or chills and states that besides the
situation with her wrist, she feels well. Looking at her chart and speaking with her you
find her only medication is lisinopril for hypertension. She has no other medical problems
of which she is aware. Physical examination of the left forearm reveals a shallow,
healing, 2.5 cm long laceration. Extending approximately 2 cm from the wound in each
direction is erythema and minute vesicles filled with clear fluid. There is no
lymphadenopathy and her temperature is normal. The most appropriate management is
to
A. prescribe acyclovir ointment
36
B. prescribe acyclovir tablets
C. prescribe cephalexin tablets
D. prescribe hydrocortisone ointment
E. recommend bacitracin ointment
Explanation:
The correct answer is D. The appearance of the skin, the history of applying neomycin
ointment, and the time course all point to an allergic contact dermatitis. Neomycin is a
very common sensitizer, with as many as 5% of the population being allergic to the
compound in some studies. Treatment for mild cases of allergic contact dermatitis is
stopping the offending agent and applying topical steroids. This case also highlights the
fact that many people wrongly assume that topical products, especially over-the-counter
products, are not medicines.
Acyclovir ointment (choice A) has been shown to be marginally effective for speeding
healing of herpes labialis, but has not been studied for extralabial sites. For a large area
of herpes vesicles, oral acyclovir would be the preferred treatment.
Acyclovir tablets (choice B) would be reasonable if this was a herpetic outbreak. The
classic appearance of herpes is grouped vesicles on an erythematous patch or
edematous plaque. The localization of herpetic lesions to the forearm would be
unusual, as would the subjective symptom of pruritus since herpetic lesions are usually
painful. Lack of lymphadenopathy would be another factor that would militate against a
diagnosis of herpes. Definitive diagnosis of herpes would be either positive culture or
identifying multinucleated cells on a Tzanck prep.
Cephalexin (choice C) is a treatment for impetigo, particularly if it is widespread. Also, if
you suspected the patient had cellulitis, systemic antibiotics would be indicated.
Clinically cellulitis would have erythema around the wound, but fever and/or
lymphadenopathy would be expected. In addition, cellulitis does not present with
vesicles nor with pruritus.
Bacitracin ointment (choice E) may be a reasonable choice if you suspected impetigo,
but that entity consists of erythematous plaques with a prominent scale-crust. This crust
is classically described as honey-colored. Impetigo generally does not exhibit vesicles.
It would also be uncommon to develop in an area being cleaned and having
antibacterial ointment applied twice a day. Note, the lack of fever and the lack of
lymphadenopathy are entirely consistent with impetigo. Another reason bacitracin would
be a poor choice in this situation is a high percentage of patients with contact sensitivity
to neomycin will cross-react to bacitracin, thereby aggravating the contact dermatitis.
A 32-year-old woman comes to the office because of "hair loss." Even though she has
not noticed much of a difference, her friends have been telling her that her "gorgeous
thick hair seems to be getting a bit sparse" lately. She states that she only notices the
difference when she looks at pictures of herself that were taken 1 years ago. She is
generally healthy and has no other complaints. She exercises 5 to 6 days a week, eats a
"low fat, low protein" diet, does not smoke cigarettes, and drinks a glass of red wine with
dinner. Her temperature is 37.0 C (98.6 F), blood pressure is 110/70 mm Hg, and pulse
is 55/min. Physical examination shows diffuse thinning of the hair and a visible scalp. At
this time, the most correct statement about her condition is:
37
A. The diffuse hair is unrelated to trichotillomania
B. The diffuse hair loss is unrelated to her "low fat, low protein" diet
C. Thyroid dysfunction is not the cause of her diffuse hair loss
D. She has an autoimmune condition called alopecia areata, in which
lymphocytes attack the hair follicle
E. She should discontinue her vigorous exercise program and the hair will regrow
Explanation:
The correct answer is A. Trichotillomania is a form of obsessive compulsive disorder
where individuals pull out large amounts of their own hair. It is usually concentrated in
certain areas, particularly above the ear (because this may be the most convenient area
to pull), and leads to patchy hair loss. Diffuse thinning of the hair is not the typical
pattern.
It is incorrect at this time to say that her diffuse hair loss is unrelated to her "low fat, low
protein" diet (choice B) because diffuse thinning of the hair is associated with many
vitamin and mineral deficiencies such as zinc, iron, and biotin deficiency. It is also seen
in some individuals with protein deficiency.
Hyperthyroidism and hypothyroidism (choice C) are associated with diffuse thinning of
the hair. Many individuals with thyroid dysfunction are asymptomatic, and therefore, you
cannot conclude that she does not have thyroid dysfunction until you do the appropriate
thyroid tests (thyroid stimulating hormone levels).
Alopecia areata is an autoimmune condition where lymphocytes attack the hair follicle
(choice D) and is typically associated with well-circumscribed areas of hair loss, not
diffuse thinning.
At this time it is not appropriate to tell her that her hair will grow back if she discontinues
her vigorous exercise program (choice E). Her diffuse hair loss can be due to many
different things, such as vitamin and mineral deficiencies, protein deficiency, thyroid
dysfunction, and HIV infection. It is possible that the exercise is associated with her
problem, but at this time, before you evaluate the other conditions, it is not appropriate
to draw this conclusion.
A 33-year-old man comes to the urgent care clinic with worsening pain of the right knee
for 2 weeks. He is an active tennis player, but cannot recall a history of trauma. He is
normally very healthy. His temperature is 37.0 C (98.6 F). A focused physical
examination confined to the musculoskeletal system is performed. The right knee is
erythematous and tender to palpation and an effusion is detected. There is normal
motion of the knee and drawer signs are negative. The remainder of the joints are
normal. The next step in management of this patient is
A. aspiration of the knee joint
B. CBC and blood culture
38
C. lyme titers
D. MRI of the knee
E. a serological test for rheumatoid factor
Explanation:
The correct answer is A. Monoarticular joint pain in a young person is infection until
proven otherwise. A complete sexual history and GU exam are essential to evaluate for
gonococcal arthritis. Given the high morbidity of an infected joint, aspiration should be
performed urgently. This result will guide antibiotic therapy. Less likely entities include
internal derangement, such as a meniscus tear or the initial presentation of an arthritis.
Evaluation with systemic labs such as a CBC and blood culture (choice B) is likely to be
negative given lack of pyrexia.
Lyme disease (choice C) may present with a migratory polyarthritis in patients from the
New England region and other infested areas. There is usually a history of outdoor
activity allowing the patient to become bitten by the Ixodes tick which transfers the
spirochete Borrelia burgdorferi to the patient. In cases of Lyme disease, the rash of
erythema migrans is often noticed several weeks or even months before arthritic
symptoms, typically, symptoms of the knee are noted. In this case, a rapidly destructive
bacterial infection of the joint must be excluded long before Lyme titer results return
from the laboratory.
An MRI of the knee (choice D) is useful to exclude internal derangement such as a
ligamentous sprain or tear, meniscal injury, fracture, or contusion. Given the erythema
of the knee, but a lack of motion pain or limitation, infection is much more likely. A knee
effusion or bone edema on MRI would suggest infection, but arthrocentesis is
necessary immediately.
Serological evaluation for arthritis (choice E) such as rheumatoid factor or HLA-B27
positivity should be performed once infection is excluded with an arthrocentesis and
internal derangement is excluded with an MRI.
A 72-year-old man comes to the office because of "rectal pain and bleeding" for the past
few weeks. The pain is relatively constant and is not associated with defecation. The
bleeding is intermittent and he thinks that it is present most of the time because he finds
bright red blood on his undergarments. He has also been feeling a "bit weak" lately and
has lost 10 pounds, but he attributes this to "getting old." He denies any change in bowel
habits. He recently moved to your town to live with his daughter and he tells you that he
has a history of hemorrhoids and that he has been eating a high fiber diet and taking
stool softeners, just as the previous physician instructed him to. Physical examination
shows a 1.3 cm blue-black partially raised, ulcerated lesion, just above the anal verge.
Rectal examination reveals guaiac-negative hard, brown stool. Anoscopy is
unremarkable. The most appropriate next step is to
A. admit him to the hospital for the evaluation for rectal carcinoma
B. advise him to take sitz baths, apply a topical steroid cream, and increase his
fiber intake
C. incise the lesion, evacuate the thrombus, and apply compression to the
incised area
39
D. perform an immediate rubber band ligation and tell him that it will undergo
degeneration
E. perform an excisional biopsy of the lesion
F. prescribe a bowel preparation and schedule a colonoscopy for next week
Explanation:
The correct answer is E. This patient most likely has malignant melanoma. The features
that make this lesion suspicious are its blue-black color, partially raised, asymmetrical
pattern, and the size (>6 mm). The first part of the diagnosis of any melanoma is by
visual inspection. Any lesion suspicious for a melanoma should be biopsied. The
preferred method is by excisional biopsy, which is full thickness. Avoidance of shave
biopsy, curettage or incisional biopsy is recommended for any lesion thought to be
melanoma because it will alter the final histological evaluation of the tumor. While the
anus is not one of the most common sites for melanoma, it can occur anywhere and
should always be part of your differential diagnosis of a suspicious lesion.
It seems that this patient may have malignant melanoma that should be evaluated with
a biopsy and it is therefore unnecessary to admit him to the hospital for the evaluation
of a rectal carcinoma (choice A) at this time. He does complain of rectal bleeding,
however he has not had a change in bowel habits and his rectal examination and
anoscopy were unremarkable. At this time, a biopsy is the best next step.
Sitz baths, application of a topical steroid cream, and increasing his fiber intake (choice
B) are the usual treatment for symptomatic external hemorrhoids which often present
with rectal pain and bleeding. Physical examination and anoscopy will often show the
hemorrhoids. Increased fiber intake and possibly stool softeners are part of the longterm management of hemorrhoids. This patient may have a malignant melanoma and
therefore requires evaluation with a biopsy.
Incise the lesion, evacuate the thrombus, and apply compression to the incised area
(choice C) is the treatment for a thrombosed external hemorrhoid. Pressure by
compression is usually all that is needed to control the bleeding. The typical
presentation of a thrombosed external hemorrhoid is an acute onset of very severe
perianal pain, particularly when walking and sitting. An external hemorrhoid arises
below the dentate line and appears as a tender blue swelling at the anal verge.
Rubber band ligation (choice D) is the treatment for refractory, symptomatic internal
hemorrhoids. Local anesthesia is given, an anoscope is inserted, and a rubber band is
placed around the hemorrhoid. The hemorrhoid will eventually become necrotic and
slough over time. Internal hemorrhoids arise above the dentate line. This patient has a
suspicious lesion near the anal verge that requires biopsy evaluation.
Bowel preparation and colonoscopy (choice F) are not the next step in management of
a lesion that is suspicious for a malignant melanoma. A biopsy needs to be performed
to establish a diagnosis.
A 12-year-old boy is brought to the office because of a 2-week history of pain and
swelling of his right leg. The pain is worse on the shin, just below the knee, and is
exacerbated by running, jumping, and going up and down stairs. His mother states that
he is a very active child; “he is always on the go." He plays basketball with friends on the
court in their backyard, and baseball on a neighborhood little league team. He recently
started playing volleyball in gym class. Physical examination reveals a pubescent boy
40
with point tenderness and swelling over the right tibial tubercle. The left leg is
unremarkable. The most likely diagnosis is
A. "growing pains"
B. Legg-Calve-Perthes disease
C. Osgood-Schlatter disease
D. osteosarcoma
E. slipped capital femoral epiphysis
Explanation:
The correct answer is C. This patient most likely has Osgood-Schlatter disease. The
typical patient is a physically active pubescent boy who has pain and swelling over the
tibial tuberosity. The pain is exacerbated by physical activity. It is caused by
apophysitis, (inflammation of the tibial tuberosity), and cartilage detachment. Diagnosis
is made by history and tibial tuberosity tenderness. Treatment is the reduction of
physical activity.
"Growing pains", (choice A) are usually characterized by deep, severe, bilateral, diffuse
pains that are worse at night. There is no associated limp.
Legg-Calve-Perthes disease (choice B), avascular necrosis of the femoral head, is
characterized by hip and knee pain, a limp, and decreased range of motion. It usually
affects boys between 4 to 8 years of age. Casting and surgery are treatment options.
Osteosarcoma (choice D), is the most common malignant bone tumor in kids. It occurs
in the metaphyses of long bones, and presents with pain, swelling, and a palpable
mass. X-rays show a lytic lesion with a "sunburst" pattern. Treatment is surgery.
Slipped Capital Femoral Epiphysis (choice E), is a disorder of overweight boys that is
caused by a displacement of the femoral head from the femoral neck. There is knee or
thigh pain, and a limp. Treatment is immediate surgical fixation.
A 12-year-old boy is brought to the office because of a 2-week history of pain and
swelling of his right leg. The pain is worse on the shin, just below the knee, and is
exacerbated by running, jumping, and going up and down stairs. His mother states that
he is a very active child; “he is always on the go." He plays basketball with friends on the
court in their backyard, and baseball on a neighborhood little league team. He recently
started playing volleyball in gym class. Physical examination reveals a pubescent boy
with point tenderness and swelling over the right tibial tubercle. The left leg is
unremarkable. The most likely diagnosis is
A. "growing pains"
B. Legg-Calve-Perthes disease
C. Osgood-Schlatter disease
D. osteosarcoma
E. slipped capital femoral epiphysis
41
Explanation:
The correct answer is C. This patient most likely has Osgood-Schlatter disease. The
typical patient is a physically active pubescent boy who has pain and swelling over the
tibial tuberosity. The pain is exacerbated by physical activity. It is caused by
apophysitis, (inflammation of the tibial tuberosity), and cartilage detachment. Diagnosis
is made by history and tibial tuberosity tenderness. Treatment is the reduction of
physical activity.
"Growing pains", (choice A) are usually characterized by deep, severe, bilateral, diffuse
pains that are worse at night. There is no associated limp.
Legg-Calve-Perthes disease (choice B), avascular necrosis of the femoral head, is
characterized by hip and knee pain, a limp, and decreased range of motion. It usually
affects boys between 4 to 8 years of age. Casting and surgery are treatment options.
Osteosarcoma (choice D), is the most common malignant bone tumor in kids. It occurs
in the metaphyses of long bones, and presents with pain, swelling, and a palpable
mass. X-rays show a lytic lesion with a "sunburst" pattern. Treatment is surgery.
Slipped Capital Femoral Epiphysis (choice E), is a disorder of overweight boys that is
caused by a displacement of the femoral head from the femoral neck. There is knee or
thigh pain, and a limp. Treatment is immediate surgical fixation.
An 18-year-old woman comes to the clinic because of a 4-month history of a "red rash"
on her elbows, knees, and around her "belly button." She noticed the lesions during the
winter, but was not particularly concerned because they were covered up by pants and
long sleeves. Now it is summer and she is too embarrassed to wear shorts or a bathing
suit. She has no significant past medical history, is up-to-date on her immunizations, and
has not traveled recently. She takes no medications and has no known allergies. She
tries to avoid all sun exposure because she tends to "burn, not tan." Physical
examination shows erythematous plaques on her elbows, knees, and umbilicus. There is
a silvery scale covering the majority of each lesion that bleed when you scrape it. The
remainder of the examination is unremarkable. The most appropriate next step is to
A. advise her to avoid sun exposure, especially direct sunlight on the lesions
B. biopsy each lesion and send for histologic evaluation
C. inject each lesion with a medium-potency corticosteroid
D. prescribe a medium-potency corticosteroid ointment and topical calcipotriene
E. refer her to a dermatologist
Explanation:
The correct answer is D. This patient most likely has psoriasis, which is a relatively
common skin condition that affects approximately 2% of the population. Psoriasis is
characterized by erythematous, thickened plaques with a silvery scale. Scraping of the
scale may lead to pinpoint bleeding, often called the Auspitz sign. The diagnosis is
usually made by physical examination. The treatment for localized lesions typically
begins with a topical corticosteroid and topical calcipotriene (a vitamin D analog that
enhances normal keratinization and inhibits epidermal cell proliferation). A topical
corticosteroid can also be used with a coal tar product, which possibly suppresses
42
epidermal DNA synthesis, as a first-line therapy. These can be tapered if the lesions
subside. If this in ineffective, corticosteroid and anthralin or tazarotene therapy
combined with ultraviolet phototherapy should be considered. If this is still ineffective,
the patient should be sent to a dermatologist.
It is inappropriate to advise her to avoid sun exposure, especially direct sunlight on the
lesions (choice A) because sunlight is thought to be helpful in treating psoriasis. She
should be encouraged to obtain natural sunlight exposure for a few minutes a day. But
she needs to be advised that unaffected areas should be covered with a sunscreen.
While psoriasis can be diagnosed by a biopsy, it is inappropriate to biopsy each lesion
and send for histologic evaluation (choice B). The diagnosis can usually be made based
on clinical presentation, especially when the lesions are classic, as they are in this
case.
It is inappropriate to inject each lesion with a medium-potency corticosteroid (choice C)
at this time. This is usually reserved for psoriasis that is not controlled by topical
therapy.
It is not necessary to refer her to a dermatologist (choice E) at this time. Any physician
should be able to recognize and treat localized psoriasis. If the treatments are not
effective in controlling her disease, you should refer her to a dermatologist.
A 6-year-old boy is brought to the office by his parents who are concerned because he
has been refusing to use his left arm for 1 day. The parents report that he has been in
good health and has not suffered any recent falls or injuries to the arm that they are
aware of. The father does recall one incident 2 days ago when he pulled upward on the
boy's right arm to prevent him from tripping as they descended a flight of stairs. The boy
is holding his right arm with the elbow flexed and the forearm pronated. He begins to cry
when you attempt to examine the arm. The most appropriate next step is
A. closed reduction and cast immobilization
B. a CT of the right arm
C. measurement of compartment pressure of the forearm
D. supination of the forearm with the elbow flexed
E. a trial of compressive bandage on the right arm
Explanation:
The correct answer is D. This boy has the signs and symptoms of a very common injury
in little children. "Nursemaid's elbow" or subluxation of the radial head, which occurs
when there is longitudinal traction on the pronated extended elbow. This traction can
cause the radial head to slip from beneath the annular ligament. Children will often
refuse to move the arm following this injury. X-rays of the forearm and elbow are usually
normal and treatment involves supination of the forearm with the elbow flexed.
Recurrence is uncommon and no immobilization of the affected arm is necessary.
Closed reduction and cast immobilization (choice A) is necessary for fractures of the
forearm, not for subluxation of the radial head. Fractures of the forearm are also very
common in children, but usually have an antecedent history of fall or injury.
Obtaining a CT of the right arm (choice B) is not necessary for this patient. X-rays of the
43
right arm can be done to rule out a fracture or hemarthrosis. A CT of the extremity is
sometimes performed if the x-ray demonstrates a fracture. The history given by the
boy's parents and the boy's presenting symptoms strongly suggest subluxation of the
radial head. X-rays of the forearm and elbow are usually normal in this setting and are
not necessary to make the diagnosis.
There is no reason to suspect compartment syndrome (choice C) in this child. Signs of
compartment syndrome include severe pain in the limb especially with active or passive
stretching of the ischemic muscle, cyanosis, and numbness. This syndrome is an
uncommon complication of any traumatic injury to the extremities. Subluxation of the
radial head is not associated with compartment syndrome.
A trial of a compressive bandage (choice E) is not necessary in this patient. Supination
of the forearm should reduce the subluxation of the radial head. Compressive bandages
or immobilization are not necessary after reduction of the subluxation.
You are seeing a 23-year-old woman in your office for a follow up visit. She had
presented for an initial visit a month ago complaining of swelling in her fingers. Today,
she continues to describe edema and erythema of her metacarpophalangeal joints. She
has also had some edema of her left elbow. She explains that her morning stiffness is
lasting more than 1 hour. She is currently on no medications and has no allergies to
medications. Her mother and maternal aunt have a history of severe rheumatoid arthritis.
Her temperature is 37.2 C (99.0 F), blood pressure is 123/65 mm Hg, pulse is 76/min,
and respirations are 18/min. She has edema and erythema of the metacarpophalangeal
joints of both hands. The remainder of her joint examination is unremarkable. Her breath
sounds are clear, and her cardiac rhythm is regular. The results of routine laboratory
studies sent during her previous visit are consistent with a diagnosis of rheumatoid
arthritis. The most appropriate pharmacologic intervention at this time to alter the course
of her disease is
A. aspirin
B. 5-fluorouracil
C. methotrexate
D. penicillamine
E. sertraline
Explanation:
The correct answer is C. It is reasonable to start the patient on a trial regimen of
methotrexate since studies have shown that disease modifying antirheumatic drugs
(DMARD) such as methotrexate have improved outcomes with rheumatoid arthritis, and
may alter the course of her disease. An NSAID or COX-2 specific inhibitor should also
be used.
Aspirin (choice A) is a first-time therapy that can help with the signs and symptoms of
her disease but will have minimal effect on the progression of disease.
5-fluorouracil (choice B), an antineoplastic agent, has no proven benefit with
rheumatoid arthritis.
Penicillamine (choice D), while being a disease modifying antirheumatic drug (DMARD),
44
is associated with too many toxic side effects to be chosen as a first-line DMARD.
Sertraline (choice E), a selective seratonin reuptake inhibitor (SSRI) used for the
treatment of depression, has no role in the management of rheumatoid arthritis.
A 68-year-old retired firefighter comes to the clinic because of pain in his left knee.
Despite being retired, he has remained very active playing tennis and golf 4 times a
week. He has been unable to play tennis for the last month because of his knee pain. In
fact, he says “sometimes when I'm walking, my knee just locks up and I can't move it at
all!” His past medical history includes hypertension, gout, and a cholecystectomy 10
years ago. His medications include an antihypertensive and an occasional nonsteroidal
antiinflammatory drug for gout flares in his ankle. Physical examination reveals full range
of motion in the left knee and moderate pain along the medial aspect of the knee. There
is a small effusion, but no erythema. Anterior and posterior drawer tests demonstrate
stability of the knee. An x-ray of the left knee shows moderate degenerative changes
with medial joint compartment narrowing, small osteophytes, and a small effusion. The
most likely cause for this patient's symptoms is
A. an anterior cruciate ligament tear
B. gout
C. iliotibial band syndrome
D. a medial collateral ligament tear
E. a medial meniscus tear
Explanation:
The correct answer is E. The two most common causes of meniscal tears are due to
traumatic injury (often seen in athletes) and degenerative processes (seen in elderly
patients who have more brittle cartilage). Individuals who experience a meniscal tear
usually experience pain and swelling as the primary symptoms. Another common
complaint is joint “locking”, or the inability to completely straighten the joint. This is due
to a piece of the torn meniscus physically impinging the joint mechanism of the knee.
An x-ray of the knee may either be normal or it may demonstrate degenerative changes
in the knee joint.
An anterior cruciate ligament tear (choice A) is not likely the cause of this patient's
symptoms. The ACL is most commonly injured during sporting activities when an
athlete suddenly pivots causing excessive rotational forces on the ligament. Other
mechanisms that can cause an ACL tear include severe trauma and work injuries.
Individuals who experience ACL tears usually describe a feeling of the joint “giving out”
and people often report hearing a “pop”. An anterior drawer test will elicit instability of
the knee joint such that the tibia is able to slide abnormally forward relative to the femur.
Gout (choice B) is not likely the cause of this patient's symptoms. Gout is an
inflammatory arthropathy that causes pain, swelling, and erythema in the affected joint.
The sensation of the knee “locking” indicates some type of internal derangement of the
knee as opposed to an inflammatory process.
Iliotibial band syndrome (choice C) is not likely the cause of this patient's symptoms.
This syndrome manifests as pain along the lateral aspect of the knee and is often seen
45
in runners. The iliotibial band represents the tensor fascia lata muscle, which runs along
the lateral aspect of the thigh and attaches to the lateral aspect of the tibia. The
ligaments of the iliotibial band contribute fibers to the lateral collateral ligament. Lateral
knee pain is due to irritation and inflammation of the distal portion of the iliotibial band
at, or just distal to, the point at which it crosses the lateral femoral epicondyle. It is a
type of overuse injury caused by repetitive friction of the iliotibial band across the lateral
femoral epicondyle.
A medial collateral ligament tear (choice D) is not likely the cause of this patient's
symptoms. The MCL is rarely injured in isolation. It usually is part of the “unhappy triad”,
used to describe injuries to the medial meniscus, anterior cruciate ligament, and the
medial collateral ligament. This triad is seen in sports such as football, when the player
is hit on the outside of the knee.
A 56-year-old woman comes to the office complaining of pain in the fingers of both of her
hands. The pain is accompanied by stiffness, is worse in the mornings, and gradually
improves throughout the day. Over the past few months she has also noticed some
"lumps" on her forearms. Her past medical history is otherwise remarkable for mild
hypertension and recent menopause. Her medications are only estrogen/progesterone
for hormone replacement. Physical examination shows tenderness of the wrists and
proximal joints of the second and third fingers, bilaterally. There are small, mobile
nodules over the extensor surfaces of her forearms and hands. Laboratory studies will
most likely show
A. decreased serum calcium levels
B. an elevated erythrocyte sedimentation rate (ESR)
C. an elevated mean corpuscular volume
D. hypercholesterolemia
E. polycythemia
Explanation:
The correct answer is B. The patient has rheumatoid arthritis. She has all of the
classical findings, which include morning stiffness, polyarticular involvement, and
rheumatoid nodules. The etiology of RA is unknown but it is a progressive, destructive
disease. The disease is seen in women at least two times more than men. It has a
prevalence of nearly 10% worldwide in persons over the age of 65. Although most
patients are rheumatoid factor positive, their levels do not correlate with the clinical
severity of the disease. More consistently, ESR is elevated during active periods of the
disease as well as during periods of remission.
Decreased serum calcium levels (choice A) are not characteristic of RA. Although
patients at this age have issues concerning bone density, there is no reflection on the
serum calcium levels. Total calcium levels may be abnormal however, but serum
calcium is often unchanged.
Because anemia of chronic disease is a normocytic or microcytic anemia, the MCV
(choice C) will be reduced, not elevated.
Hypercholesterolemia (choice D) is not characteristic of RA, but is a common clinical
46
finding in nephritic syndrome. This patient's age and postmenopausal status make her
more likely to have high cholesterol compared to a younger, menstruating woman, but
this is not necessarily the case.
Most patients with moderate RA suffer from anemia of chronic disease so they will not
be polycythemic (choice E).
A young mother brings in her 3-year-old daughter to the clinic because of worsening
acne problems on her daughter's face. She had a normal birth without complications.
However, she is developmentally slow and has had a few episodes of seizures of
unknown etiology. Her medications include phenytoin and a multivitamin. The child is
playful and her size and weight are appropriate for her age. She has numerous firm,
flesh-colored papules scatter over her nose, both cheeks, and chin. There are no
pustules or inflammatory papules. She also has multiple, small, hypopigmented ash leaf
and confetti-type macules on both lower extremities. The rest of her examination is
unremarkable. The next most appropriate step is to
A. check morning level of cortisol
B. check serum level of DHEAS and testosterone
C. prescribe oral minocycline
D. prescribe topical benzoyl peroxide every morning and topical erythromycin
every evening
E. send her for an x-ray of the skull
Explanation:
The correct answer is E. An x-ray of the skull is the most appropriate next step,
because this patient has tuberous sclerosis (TS). TS is also known as epiloia (epi =
epilepsy, loi = low intelligence, a = adenoma sebaceum). This is an autosomal
dominant syndrome marked by other features including periungual fibromas, shagreen
plaques, oral papillomatosis, ash leaf hypomelanotic macules, skin fibromas, and café
au lait spots. Adenoma sebaceum are 1-3 mm, yellowish, red discrete, waxy papules
that are distributed symmetrically over the cheeks, nose, and forehead. Histologically,
these are angiofibromas. Congenital hypomelanotic macules are found in 85% of TS
patients which may be detected at birth in most patients. They may be shaped like an
ash leaf, but linear and confetti-type white macules may be present. The hypomelanotic
macules are an indication for a skull x-ray evaluation to search for calcified intracranial
nodules. These are hamartomatous proliferations of glial and neuronal tissue produce
potato-like nodules or brain stones in the cortex leading to seizures.
Checking levels of cortisol (choice A) and DHEAS/ testosterone (choice B) is incorrect
because this patient does not show other signs of endocrine abnormalities (i.e.,
hypertrichosis and enlarged external genitalia).
Prescribing minocycline (choice C) and benzoyl peroxide and erythromycin (choice D)
is incorrect, because these treatments are aimed at treating acne lesions.
A 51-year old woman with systemic lupus erythematosus comes to the clinic with right
47
hip pain for the past 3 days. She has no history of trauma. The patient has been
postmenopausal for the past 3 years. She takes naproxen, ibuprofen, and prednisone
100 mg a day. She has been taking these medications for the past 10 years. She tells
you that she stopped taking estrogen secondary to occasional nausea. Her temperature
is 37.4 C (99.4 F). Physical examination reveals a butterfly facial rash, multiple swollen
joints of the hands and feet, and a systolic flow murmur. A plain radiograph of the hip is
normal. A bone scan reveals decreased 99Tc MDP tracer uptake in the right femoral
head. The patient should be advised to
A. avoid weightbearing and immediately stop prednisone
B. avoid weightbearing and taper off prednisone
C. begin an exercise program and calcium supplementation
D. begin an exercise program and taper off prednisone
E. begin an exercise program, calcium supplementation, and estrogen
replacement therapy
Explanation:
The correct answer is B. The history, physical exam, and bone scan results are
consistent with systemic lupus erythematosus and avascular necrosis (AVN) of the right
femoral head. This is a common complication of long-term steroid therapy. AVN may
also be a result of pancreatitis, fat embolus, alcoholism, sickle cell anemia, air emboli,
or idiopathic causes as in Legg-Calve-Perthes disease. Avoidance of activity, especially
weightbearing is necessary to ensure that the femoral head does not collapse and
fracture. Given a prednisone dose greater than 10mg QD, a slow taper should be
initiated to allow the adrenal glands to resume normal cortisol production.
A rapid taper of prednisone (choice A) could result in adrenal insufficiency. Symptoms
of this iatrogenic condition include fatigue, tiredness, depression, and weight loss. More
severe cases result in hypotension, irregular pulse, and nausea.
Exercise involving the lower extremities (choice C) must be avoided as collapse and
fracture of the femoral head may occur. AVN is not a symptom of osteoporosis, so
standard osteoporosis treatments do not apply.
Exercise involving the lower extremities (choice D) must be avoided as collapse and
fracture of the femoral head may occur.
Exercise involving the lower extremities (choice E) must be avoided as collapse and
fracture of the femoral head may occur. AVN is not a symptom of osteoporosis, so
standard osteoporosis treatments do not apply. Since this patient is postmenopausal,
estrogen replacement therapy should be begun as a separate issue.
A 44-year-old Asian man comes to the office for a health maintenance examination. He
does not take any medications routinely and has no diagnosed medical problems. He
does not smoke or drink alcohol on regular basis. On several occasions, he has tried
diet and exercise to shed some weight without success. He is an obese male with
hyperhidrosis. There is hyperpigmentation with a velvety appearance on the nape and
bilateral axillae. He has similar lesions in the groin area. Oral mucosa and palmoplantar
surfaces are unremarkable. The abnormal laboratory test result that is most likely
correlated with these findings is an elevated
48
A. chorionic embryo antigen
B. hemoglobin A1C
C. testosterone
D. total cholesterol
E. triglyceride
Explanation:
The correct answer is B. Acanthosis nigricans (AN) is characterized by
hyperpigmentation and papillary hypertrophy, which are symmetrically distributed.
Different types of AN have been described. The most common variety of AN is grayish,
velvety thickening of the skin of the sides of the neck, axillae, and groin. It occurs in
obesity, insulin-resistance, with or without endocrine disorders such as diabetes
mellitus and hypothyroidism as well as Cushing's syndrome and Addison's disease.
Therefore, an elevated hemoglobin A1C, which is associated with diabetes, is
correlated with these findings.
High chorionic embryo antigen (choice A) is incorrect. One of the rare variants of AN is
the malignant type which may either precede, accompany, or follow the onset of internal
cancer. Most cases are associated with adenocarcinoma, especially of the
gastrointestinal tract, lung and breast. Less often, the gallbladder, pancreas,
esophagus, liver, prostate, kidney, colon, rectum, uterus, and ovaries are involved.
High testosterone level (choice C) is incorrect. Sex hormones has not been associated
with AN.
High total cholesterol (choice D) and high triglyceride (choice E) are both incorrect.
Although, these two findings are likely in an obese patient, they are not directly related
to AN.
A 35-year-old elementary school art teacher comes into the office worried about a
growing lesion on her left thumb over the last 3-4 weeks. She recalls working with sharp
knives and accidentally stabbing the thumb in sculpture class around the same time she
noticed the lesion. She tells you that initially the lesion looked liked a small "pimple"
which grew in size, bleeding when irritated, and has rough surfaces. She describes a
"pulsing" tender sensation. Physical examination shows a 1.2 cm x 1.0 cm polypoid, soft,
red mass constricted by a collarette at the base of the lesion located on the lateral nail
fold of the left thumb. No pulse over the lesion is appreciated on palpation. The
remainder of the cutaneous examination is unremarkable. The most appropriate next
step is to
A. check HIV status because this is most likely Kaposi sarcoma
B. shave and electrodesiccate the base of the lesion and send the specimen to
pathology for analysis
C. shave the lesion off and throw it away because it is a benign lesion
D. tell her this is an acquired hemangioma and it will involute on its own
49
E. tell her this is a fungal infection and you need to treat it with oral antifungal
agents
Explanation:
The correct answer is B. A pyogenic granuloma is a result of exuberant granulation
tissue following minor trauma (usually with sharp objects) or infection of the skin. It
presents as an intensely red, sometimes ulcerated, pedunculated mass that bleeds
easily on provocation. Treatment usually involves removal of the mass by either shave
and electrodessication or complete excision. Clinically, it can resemble amelanotic
melanoma; therefore, always submit the specimen for histologic confirmation.
Kaposi sarcoma (KS) (choice A) is incorrect because HIV-associated lesions of KS are
generally asymptomatic and frequently seen on the soles of feet, the hard palate, and
the tip of the nose. Other common sites include face, trunk, and penis. In this setting it
occurs predominantly among homosexual men. It is associated with an increased
number of sexual partners, a prior history of oropharyngeal gonorrhea, and increased
contact with a sex partner's feces through oral-anal contact.
Shave the lesion off and throw it away because it is a benign lesion (choice C) is
incorrect because you should never shave a lesion without evaluating it for histologic
confirmation. There have been cases reported of amelanotic melanomas that were
removed with a strong clinical suspicion of pyogenic granuloma.
Acquired hemangiomas, also known as cherry angiomas, (choice D) is incorrect
because these lesions are small, bright-red, papules composed of ecstatic vessels.
They generally arise in middle age, usually on the abdomen. They tend not to grow
rapidly in size and do not involute spontaneously.
Fungal infection (choice E) is incorrect because infection of the nails tend to result in
dystrophic, thickened, discolored nail plates with generalized erythema and scaling of
the proximal nail fold.
A 66-year-old woman comes to the clinic because of a 3-month history of increasing
pain in her right knee that began after a very long car trip across the country. She has a
severe, sharp, non-radiating pain primarily in the medial, but also extending to the
inferior aspect of her right knee. It occurs anytime of the day or night and sometimes
awakens her from sleep. There is no activity that decreases the pain or initiates it. She
says she also "loses feeling" in her right lateral thigh sometimes and there is often a
"throbbing feeling" in different parts of her right leg. She denies urinary problems or
symptoms in the left leg. She takes ibuprofen for the pain even though it is "not very
effective." Physical therapy and deep massage have "not worked and have even made
things worse." She had right leg pain 15 years earlier, which was treated with a lumbar
laminectomy and fusion. She does not remember exactly what the original pain felt like,
but since the surgery it has been much better. There was an episode of increased pain 5
years ago, but this resolved on its own. She had seen a different physician 5 weeks ago
for this knee pain who sent her for an MRI of the lumbar spine, which showed L4-5
stable appearing spondylolisthesis and no significant root or cord pathology. She also
had bilateral knee x-rays at that time, which showed mild to moderate arthritic changes.
Physical examination shows a normal motor exam, loss of pinprick sensation over most
of the right lateral thigh, tenderness to palpation of the right femoral nerve and to deep
palpation of the medial knee. Her reflexes are all normal and symmetric. The most
appropriate next step in the management of this patient is to
50
A. give her a prescription for morphine
B. order an MRI of the thoracic spine
C. refer her for neurosurgical evaluation
D. refer her for orthopedic evaluation
E. repeat the lumbar spine MRI
Explanation:
The correct answer is D. This patient presents with localized knee pain, which is
exacerbated by palpation of the medial knee. It is most likely due to structural knee
pathology, perhaps of the medial meniscus. There are no elements of the history or
exam that suggest that the pain is radicular. The right lateral thigh symptoms are
probably due to femoral nerve entrapment and warrant no clinical intervention if they
are not bothersome.
Merely treating the pain without a proper evaluation and diagnosis would be a mistake
(choice A). Structural knee pathology is often treatable surgically.
An MRI of the thoracic spine (choice B) or lumbar spine (choice E) are not indicated as
there is no evidence of spine or root pathology.
Since there is no reason to suspect a new neurosurgical problem based on the above
explanation, neurosurgical consult is not indicated (choice C).
51